You are on page 1of 108

CAMPUS RECRUITMENT

COMPLETE REFERENCE

LOG-ON
to
www.CampusRecruitment.co.in

Praxis Groups
PRAXIS GROUPS
Indias First online educational material publishers

Address:
Mig-B-35,
Indian Airlines Colony,
Begumpet, Hyderabad-500003.
Phone: 040-40203585, 09246844022
Email: contact@campusrecruitment.co.in
Website: www.campusrecruitment.co.in

Third Book Edition 2015


Online Edition 2007, 2008, 2009, 2010

ISBN 978-81-930685-1-9

Copyright 2006 -2015 Praxis Groups. All Rights Reserved


All rights reserved. No part of this book may be reproduced in any form through any electronic or mechanical means
(including photocopying, recording, or information storage and retrieval) without prior permission in writing from
the publisher, except for reading and browsing via the World Wide Web.

Information contained in this book has been published by Praxis Groups and the content has been obtained and
developed by its authors from sources believed to be reliable and correct to the best of their knowledge. However, the
Publisher and the Authors shall, in no event, be liable for any errors, omissions or damages arising out of the use of
this information and specifically disclaim any implied warranties or merchant ability or fitness for any particular use.
Disputes of any are subject to Hyderabad Jurisdiction only.

Bulk Purchases
Companies, Institutions and other organizations wishing to make bulk purchases of any book, published by PRAXIS
GROUPS should contact at -
Phone: 09246844022 , 07032206275, 040-40203585,
Email: contact@campusrecruitment.co.in
Address: Mig-B-35, Indian Airlines Colony, Begumpet, Hyderabad. Pin: 500003
About the Authors
Ramanadham Ramesh Babu, M.A. (Litt)., M.A(Edu)., B.L, (Ph.D):
Assistant Professor, Dept. of English - GITAM University, Visakhapatnam
This book is fortunate enough to have him as an author. He has profound experience in teaching
and training the students in various national and international academic and competitive exams;
his passion for English made him work on innovative research and he is going to receive his
doctorate soon. He has gifted us with fabulous content for the Verbal Ability.

Israel Battu (MCA, National Institute of Technology, Calicut, Kerala):


A man with enthusiasm to contribute for the field of education which led him to establish an
institute. His expertise and enthusiasm resulted in producing outstanding material in Logical
Reasoning.

Akash R Bhutada (B.E, University of Pune, PGDA, IMT Hyderabad):


His passion for Aptitude exams and Aptitude contests produced exceptional material for
Quantitative Aptitude.
Vijaya Laxmi Krishnan (M.A PGDTE) and A. Kalyani (M.A English) :
Their vast academic experience, coupled with their interaction with students of varying abilities
has contributed significantly in the creation of this material in Verbal Ability.

About 'Technical Interview' Authors


Dr. P.Chandra Shekhar, Associate Professor, Department of ECE, OU.
Dr. Rega Rajendra, Professor in Mechanical Engineering, OU.
Ch. Siva Kumar, Assistant Professor, Department of EEE, OU.
Dr. R.Srinivasa Kumar and Dr. Suresh Kumar, Professors in Dept of Civil Engineering, OU.
Kiran Kumar Cheriyala MS(UK), B-Tech, Tech lead in Prokarma Info Tech
are esteemed professors of Osmania University. They have been visiting MNCs and conducting
interviews for the past several years.

About Quality Analyst, Co-ordinator and Executive


Vamshi Krishna Kenche (B-Tech, JNTU):
A man of commitment, honesty and hard work. He has done remarkable work in the crucial areas -
Quality Analysis, coordinating the authors/artists/data entries and finally contributing his creative
thoughts. Without Vamshi, this book wouldnt have seen the light of the day.

About Data Entry operators


Aruna sherpally and Md.Awez akram have played a very important role in managing entire work
of the book without any confusion on the computer. They have produced a quality output despite
many hurdles.

About Graphic designers


Ranjith Kumar Durishetty (B-com, BFA): Ranjith has contributed the art work for the book.
Inside the Book
Introduction to Campus Recruitment Procedure..................................................................................................... i
QUANTITATIVE APTITUDE LOGICAL REASONING
Number System...................................................... 1.1 CodingDecoding................................................. 2.1
Averages................................................................... 1.13 Seating Arrangement.............................................. 2.7
Ratio Proportion................................................... 1.19 Day Sequence/ Calender........................................ 2.12
Chain Rule / Variation............................................ 1.27 Direction Sense Test................................................ 2.20
Percentages.............................................................. 1.33 Blood Relations........................................................ 2.27
Mixtures and Allegations...................................... 1.39 Syllogisms................................................................ 2.33
Time and Work........................................................ 1.43 Series......................................................................... 2.43
Pipes and Cisterns.................................................. 1.47 Analogy Classification ....................................... 2.47
Time Speed and Distance...................................... 1.52 Alphabet Test........................................................... 2.50
Clocks....................................................................... 1.62 Character Puzzle..................................................... 2.54
Profit and Loss........................................................ 1.66 Symbol Based Operations...................................... 2.59
Interests and Discounts.......................................... 1.71 Arithmetic Reasoning............................................. 2.63
Partnerships............................................................. 1.81 Statements and Arguments................................... 2.68
Problem Solving...................................................... 1.85 Non Verbal Reasoning............................................ 2.73
Basic Geometry........................................................ 1.88 Cubes and Dices...................................................... 2.79
Solid Mensuration.................................................. 1.97 VERBAL ABILITY
Co-ordinate Geometry............................................ 1.104 Grammar
Permutations and Combinations.......................... 1.110 Sentence.................................................................... 3.1
Probability................................................................ 1.118 Parts of Speech......................................................... 3.2
Data Sufficiency....................................................... 1.129 Noun............................................................... 3.3
Data Interpretation................................................. 1.135 Pronoun.......................................................... 3.5
Tabular Data Interpretation..................... 1.135 Articles.................................................... 3.7
Bar Graphs................................................. 1.143 Verb................................................................. 3.9
Line Graphs or Cartesian Graphs........... 1.147 Tenses...................................................... 3.10
Pie Chart..................................................... 1.152 SubjectVerb Agreement...................... 3.13
Venn Diagrams.......................................... 1.155 Active and Passive Voice...................... 3.14
Net Diagram.............................................. 1.157 Adverb............................................................ 3.16
Mixed Diagrams........................................ 1.158 Adjective......................................................... 3.18
Missed Data Tables................................... 1.161 Preposition..................................................... 3.20
Inequalities............................................................... 1.165 Conjunction.................................................... 3.22
Surds......................................................................... 1.169 Reported Speech.................................... 3.23
Functions.................................................................. 1.175 Sentence Correction/ Spotting Errors................... 3.24
Logarithms............................................................... 1.181 Vocabulary................................................................ 3.34
Set Theory................................................................. 1.185 Antonyms................................................................. 3.40
Progressions............................................................. 1.189 Synonyms................................................................. 3.50
Quadratic Equations............................................... 1.193 Analogy..................................................................... 3.53
Phrasal Verbs............................................................ 3.58
Idiomatic Expressions............................................. 3.59
Reading Comprehension........................................ 3.61 GROUP DISCUSSION
Closet Fill in the blanks....................................... 3.77 Introduction............................................................. 6.41
Sentence Rearrangement........................................ 3.84 Skills required for Group Discussion................... 6.42
Theme Detection...................................................... 3.100 Steps to approach a Group Discussion................ 6.43
PLACEMENT PAPERS Reasons for failure in Group Discussion............. 6.44
Placement Paper-1................................................... 4.1 Summary................................................................... 6.44
Placement Paper-2................................................... 4.11
Placement Paper-3................................................... 4.22 How to know a company is genuine.................... 6.45
Placement Paper-4................................................... 4.33
Placement Paper-5................................................... 4.43
Placement Paper-6 to 40 visit www.CampusRecruitment.co.in
TECHNICAL INTERVIEW
Civil Engineering..................................................... 5.1
Electronics and Communication Engineering.... 5.6
Electrical and Electronics Engineering................. 5.11
Mechanical Engineering......................................... 5.18
Computer Science/ Information Technology....... 5.22
Project Interview Questions................................... 5.29
HR INTERVIEW
What qualities does an Interviewer observe?...... 6.1
Tips for Interview.................................................... 6.2
Do's and Dont's of Interview.................................. 6.2
Do's and Dont's of Body Language...................... 6.3
Answering Interview Questions........................... 6.4
Mock Interview with Body Language.................. 6.6
Reasons for failure in Interviews.......................... 6.17
TIPS FOR SEARCHING/ APPLYING JOBS
Different ways to apply for jobs............................ 6.18
Applying jobs through companies website......... 6.19
Jobs at Government/PSUs...................................... 6.21
Apply Govt/PSU jobs in Naukri/Monster............ 6.22
Selection Procedure in PSUs.................................. 6.23
List of PSUs.............................................................. 6.23
Tips to use Naukri.com.......................................... 6.26
Tips to use Monster.com......................................... 6.31
Career Path based on Academic Percentage........ 6.35
RESUME PREPARATION
Tips to build an effective Resume......................... 6.37
Things to avoid when writing Resume................ 6.37
Contents of a Good Resume.................................. 6.37
Sample Resume Format.......................................... 6.39
Introduction to Campus Recruitment Procedure
Every student chooses an educational institute on the basis of three important factors-
A. What would be the quality of education imparted?
B. How would the education benefit the student academically or otherwise?
C. The potential jobs that would be offered on the campus to the students who pass out of the institute.
The last factor is apparently the most important criteria that would be evaluated by the students in choosing a
particular academic institution. The campus recruitment procedure has become one of the most popular avenues
to recruit people into companies.
Companies hire engineering graduates into roles that range from software application development, delivery
roles, software/ hardware testing, research and development, application maintenance network security and
support etc. Management graduates are hired for roles in business development, IT consulting, business
analysis, customer relationship management, HR roles, sales, marketing, finance etc.
Before a student braces himself to face the arduous task of appearing for the campus selection procedure, there
are a few points that he has to bear in mind
1. Interact with senior students who have been hired into different organizations. Taking a cue from the
seniors would effectively help a student to know the areas that would require extensive preparation and
the ones that wouldnt.
2. Interact with the faculty and understand how the technical interviews could be facilitated.
3. Map your strengths to the profile of the job. This is extremely important because there may be scenarios
in which a student may have more than one offer at hand. Understanding the profile of the job and the
opportunities for growth within the organization, besides the CTC offered would benefit the students to
narrow down their choices.
4. Study and understand the philosophy, culture and values of the companies that recruit college
graduates.
Campus Recruitment Procedure:
Most colleges that offer campus recruitment facilitate the recruitment through a special department known as
the placement department. The placement department is steered by a placement officer who oversees the entire
recruitment process.
The various stages that are involved in a typical campus recruitment program are as follows-
Pre-placement talk
Aptitude tests
Group discussion
Technical Interview
HR Interview.
1. Pre Placement Talk:
The pre-placement talk is a presentation that is given by the recruiting companys HR and recruiting team.
Various aspects of the company such as its profile, history, milestone achievements, organizational goals, its
vision, mission, the job profile, products, services, product lines, customers, locations, branches,
organizational chart, senior management etc. The role of the job offered as well as its description along with
the selection criteria, CTC, designation etc. are also explained in detail. Generally the pre-placement talk is
presented by a senior member of the delivery or the HR team.
The general format of the selection process remains the same across companies that hire campus graduates.
Minor variation may be present. Selection happens in the following stages.
2. Aptitude Test:
Aptitude test is conducted to evaluate how effectively a student could respond to a task or a situation and
their communication skills. In short, this area tests a candidates problem solving ability. The areas that are
normally tested are numerical or quantitative ability, logical reasoning, verbal ability and data sufficiency.

www.campusrecruitment.co.in i
a. Quantitative Aptitude:
Numerical ability entails multiple choice questions that are from the topics mostly covered in high school
along with some advanced topics. The various topics from which questions may be asked are Number theory,
Averages, Ratio and proportion, Time and Distance, Percentages, mixtures and allegation, permutations,
combinations, probability etc. The purpose of this test is to assess the problem solving ability of a candidate
under constraints in time. This area can be effectively countered if a student prepares sufficiently beforehand.
b. Verbal Ability:
This area tests the communication skills, reading ability and also the grammatical knowledge of a candidate.
The type of questions that may be asked in verbal ability include grammar based questions (sentence
correction/ error identification), vocabulary based questions (para jumbles, synonyms, antonyms, fill in the
blanks, cloze passages), idioms and phrases, reading comprehension and occasionally descriptive writing
(essays, formal/informal letters, analytical/ issue writing section). It is mandatory for a candidate to have basic
rules of English in place before he or she appears for the campus placement process.
c. Analytical and Logical Reasoning:
This section tests the logical reasoning and the analytical ability of a candidate. The questions are generally
given in the form of puzzles and a set of questions follow the puzzle. It is required by a candidate to rationally
approach the puzzle by interpreting the logic. Verbal based reasoning questions such as cause and effect,
assertion and reasons may also be asked.
d. Data Interpretation and Data Sufficiency:
Data is presented in various forms such as bar graphs, pie charts and data should be interpreted accordingly.
In data sufficiency, a problem is presented with some data and a candidate has to determine if the given
amount of data is sufficient for problem solving.
The aptitude round cannot be underestimated because it is a process of eliminating candidates who do not
have enough problem solving abilities, reasoning skills or acceptable levels of communication. While a few
companies may lay more emphasis on communication and numerical abilities, a few others may stress upon
analytical abilities. Regardless of how well a candidate fares academically, he or she should prepare
sufficiently for the aptitude test as this stage in an inevitable phase of any campus selection process.
3. Group Discussion:
Those selected in the aptitude test will be called for group discussion. Group discussion is a process of
selection rather than a process of elimination. The recruiting team will evaluate certain personality traits like
confidence, communicating with the team, participation, ability to present ones views in a clear and concise
manner, interpersonal skills, leadership skills etc. These are the traits that the employers would want to see in
their potential employees. The main intention of group discussion is to assess the behavior of a candidate in a
group. In the GD round, there are usually a minimum of 5 and a maximum of 10 candidates. The topic of the
discussion is normally related current topics, hypothetical situations, problematic situations, abstract topics
etc. Students who are confident, have a clear thought process and are able to articulate their thoughts lead the
group discussion. Hence, students need to be positive, confident and dynamic in their attitude in this round.
They should also develop effective listening skills that would enable them to listen and understand others
perspective. Students are advised to keep abreast of current affairs and are expected to familiarize themselves
with the popular topics in news. They are advised to form small groups and discuss various topics which
would bolster their efforts to successively participate in the group discussions.
4. Technical Interview:
The pre final round of the selection process is the technical interview. A student appearing for the technical
round should be thorough with the fundamental aspects of his subject. While a student may not be expected
to know the entire subject inside out, he or she is expected to be proficient in the basic aspects of the subject
and able to present the subject in a well formatted manner to his interviewers. It would greatly benefit the
students if they would have completed their projects on their own rather than plagiarizing (copying) from
other sources. This would exhibit the ingenuity of a student and increase his chances of clearing the technical
round. Students who have interned in good organizations have an edge above the others in the technical
round as company internships are greatly valued by the recruiters.
www.campusrecruitment.co.in ii
5. HR Interview:
The HR interview is more of a two way process. A student is assessed for his communication, attitude,
confidence, flexibility, enthusiasm, behavioral skills etc. The company HR will market their company through
the HR round. A student should be confident, but not over confident in the HR round. He or she should be
honest and polite in answering the questions and also ask questions to the company HR regarding basic
policies, procedures and of course, the CTC. Many a time it is quite common for technically strong candidates
to fail to get through the HR round. This may happen if the candidate has poor presentation skills, is
overconfident or is rude or extremely timid in his or her attitude. Self-grooming is very important for a
candidate to clear this round of the campus selection.
The recruitment process weighs heavily on the attitude of a candidate. A candidate should exhibit positive
thinking, learnability, proper body language, confidence, clarity of thought, interpersonal skills, future goals
etc. This segment has about 75% weightage.
Academic knowledge has a weightage of about 10%. This test the students fundamental knowledge of the
core subjects in his or her branch and also its practical use.
Communication skills carry a weightage of about 10%.
Preparing a good resume is equally important while bracing for the campus selection process. The resume is
the first impression a recruiter has of the candidate. The resume should be prepared in such a way that it
reflects a candidates capabilities, his strengths, achievements and areas of interests. Hence sufficient efforts
should go into preparing a good resume. For further reading, refer to Resume section of this book.
Knowledge about current affairs and extra-curricular activities carry about 5% weightage. These areas should
also be concentrated upon to increase the chances of making it through the campus interviews.
Thus, a thorough and systematic preparation in each of the areas mentioned above would go a long way in
ensuring that a candidate gets through the right company.

www.campusrecruitment.co.in iii
QUANTITATIVE APTITUDE
NUMBER SYSTEM
CONCEPTS Note: A terminating decimal will have a finite number of
In HinduArabic system we use ten symbols 0, 1, 2, 3, 4, digits after the decimal point.
5, 6, 7, 8, 9 called digits to represent any number. This is 3 5 25
e.g.: =0.75 , =1.25 , =1.5625 .
the decimal system where we use the digits 0 to 9. Here 4 4 16
0 is called insignificant digit where as 1, . . . . . . . , 9 are Repeating Decimals: A decimal number that has digits
called significant digits. that repeat forever.
Classification of Numbers: 1
e.g.: =0.333 .. . .(here, 3 repeats forever.)
Natural Numbers: The numbers 1, 2, 3, 4, 5, 6, . . . . . . 3
which we use in counting are known as natural numbers. A decimal that neither terminates nor repeats is termed as
The set of all natural numbers can be represented by N a NonRepeating Decimal.
= {1, 2, 3, 4, 5, . . . . . . . . . .} e.g.: 2=1.4142135623 . .. .
Whole Numbers: If we include 0 among the natural Real Numbers: The rational and irrational numbers
numbers then the numbers 0, 1, 2, 3, 4, 5, . . . . are called together are called real numbers.
whole numbers. Hence, every natural number is a whole 13 2 3 +4
e.g.: , , , etc are real numbers.
number. The set of whole numbers is represented by W. 21 5 7 2
Integers: All counting numbers and their negatives The set of real numbers is denoted by R.
including zero are known as integers. Even Numbers: All those natural numbers which are
The set of integers can be represented by Z or I. exactly divisible by 2 are called even numbers.
Z = {. . . . . . 4, 3, 2, 1, 0, 1, 2, 3, 4, . . . . .} e.g.: 2, 6, 8, 10, . . . . are even numbers.
Every natural number is an integer but every integer is not Odd Numbers: An integer that can not exactly divided
natural number. by 2 is an Odd number.
Positive Integers: The set I + = {1, 2, 3, 4, . . . . .} is the set e.g.: 1, 3, 5, 7, . . . . are odd numbers.
of all positive integers. Positive integers and Natural Prime Numbers: A Prime Number can be divided
numbers are synonyms. evenly only by 1, or itself. And it must be a whole
Negative Integers: The set I = {. . . , 3, 2, 1} is the set number greater than 1.
of all negative integers. e.g.: Numbers 2, 3, 5, 7, 11, 13, 17, . . . . are prime.
0 (zero) is neither positive nor negative. All primes which are greater than 3 are of the form
Non Negative Integers: The set {0, 1, 2, 3, . . . } is the set (6n+1) or (6n1).
of all non negative integers. Note:
p 1 is not a prime number.
Rational Numbers: The numbers of the form ,
q 2 is the least and only even prime number.
where p and q are integers, p is not divisible by q and 3 is the least odd prime number.
q 0, are known as rational numbers. Prime numbers up to 100 are 2, 3, 5, 7, 11, 13, 17, 19,
(or) Any number that can be written in fraction form is 23, 29, 31, 37, 41, 43, 47, 53, 57, 61, 67, 71, 73, 79, 83,89,97.
a rational number. This includes integers, terminating There are 25 prime numbers up to 100.
decimals, and repeating decimals as well as fractions. Composite Number: Natural numbers greater than 1
3 5 5 1 3 which are not prime, are known as composite numbers.
e.g.: , , , , etc
7 2 9 2 5 The number 1 is neither prime nor composite.
The set of rational numbers is denoted by Q. Two numbers which have only 1 as the common factor
Irrational Numbers: Any real number that cannot be are called coprimes (or) relatively prime to each other.
written in fraction form is an irrational number. These e.g.: 3 and 5 are co primes.
numbers include the non-terminating, non-repeating Note:
decimals. Natural Numbers = Prime + Composite Numbers.
10 22 Whole Numbers = 0 (Zero) + Natural Numbers.
e.g.: Absolute value of , , 2 , 3 , 10 . ..
3 7 Integers = Negative Integers + 0 + Positive Integers.
Real Numbers = Rational + Irrational Numbers.

Log on to www.CampusRecruitment.co.in and ask doubt with Question Id. 1.1


Test of Divisibility: e.g.: In the number 9823, the sum of the digits at odd
Divisibility by 2: A number is divisible by 2 if the unit's places is 9+2=11 and the sum of the digits at even places
digit is either zero or divisible by 2. is 8+3=11. The difference between them is 11 11 = 0.
e.g.: Units digit of 76 is 6 which is divisible by 2 hence Hence, the given number is divisible by 11.
76 is divisible by 2. e.g.: 14641
Units digit of 330 is 0 so it is divisible by 2. 1464 1 is 1463
Divisibility by 3: A number is divisible by 3 if sum of 146 3 is 143
all digits in it is divisible by 3. 143 = 11, which is divisible by 11, so 14641 is divisible
e.g.: The number 273 is divisible by 3 since 2 + 7 + 3 = 12 by 11.
which is divisible by 3. When a number is divisible by another number,
Divisibility by 4: A number is divisible by 4, if the then it is also divisible by each of the factors of that
number formed by the last two digits in it is divisible by number.
4, or both the last digits are zeros. Divisibility by 12: A number is divisible by 12 if it is
e.g.: The number 5004 is divisible by 4 since last two divisible by 3 and 4.
digits 04 is divisible by 4. e.g.: The number 1644 is divisible by 12 as it is divisible
Divisibility by 5: A number is divisible by 5 if the units by 3 and 4.
digit in the number is either 0 or 5. Divisibility by 14: The number is divisible by 7 and 2.
e.g.: 375 is divisible by 5 as 5 is in the units place. Divisibility by 15: The number is divisible by 3 and 5.
Divisibility by 6: A number is divisible by 6 if it is Divisibility by 16: The number is divisible by 8 and 2.
even and sum of all digits is divisible by 3. Divisibility by 18: An even number satisfying the
e.g.: The number 6492 is divisible as it is even and sum divisibility test by 9 is also divisible by 18.
of its digits 6 + 4 + 9 + 2 = 21 is divisible by 3. e.g.: The number 80388 is divisible by 18 as it satisfies
Divisibility by 7: If you double the last digit and the divisibility test of 9.
subtract it from the rest of the number and the answer Divisibility by 25: A number is divisible by 25 if the
is, 0 (zero) or divisible by 7. number formed by the last two digits is divisible by 25
(Note: you can apply this rule to that answer again if or the last two digits are zero.
you want) e.g.: The number 7975 is divisible by 25 as the last two
e.g.: Consider the number 10717. digits are divisible by 25.
On doubling the unit's digit 7 we get 14. Divisibility by 88: A number is divisible by 88 if it
On omitting the unit digit of 10717 we get 1071. divisible by 11 and 8.
Now, 107114 = 1057 is divisible by 7. e.g.: The number 10824 is divisible by 88 as it is divisible
10717 is divisible by 7. by both 11 and 8.
Divisibility by 8: A number is divisible by 8, if the Divisibility by 125: A number is divisible by 125 if the
number formed by last 3 digits is divisible by 8. number formed by last three digits is divisible by 125 or
e.g.: The number 6573392 is divisible by 8 as the last 3 the last three digits are zero.
digits '392' is divisible by 8. e.g.: 43453375 is divisible by 125 as the last three digits
Divisibility by 9: A number is divisible by 9 if the sum 375 are divisible by 125.
of its digit is divisible by 9. Common Factors:
e.g.: The number 15606 is divisible by 9 as the sum of A common factor of two or more numbers is a number
the digits 1 + 5 + 6 + 0 + 6 = 18 is divisible by 9. which divides each of them exactly.
Divisibility by 10: A number is divisible by 10, if it ends e.g.: 3 is a common factor of 6 and 15.
in zero. Highest Common Factor (HCF):
e.g.: The last digit of 4470 is zero. So, it is divisible by 10. Highest common factor of two or more numbers is the
Divisibility by 11: A number is divisible by 11 if the greatest number that divides each of them exactly.
difference of the sum of the digits at odd places and sum e.g.: 3, 4, 6, 12 are the factors of 12 and 36. Among them
of the digits at the even places is either zero or divisible the greatest is 12. Hence the HCF of 12, 36 is 12.
by 11. (or) Subtract the first digit from a number made HCF is also called as Greatest common divisor (GCD) or
by the other digits. If that number is divisible by 11 then Greatest Common measure (GCM).
the original number is, too.

Log on to www.CampusRecruitment.co.in and ask doubt with Question Id. 1.2


Find the details below to purchase the book.

How to order
Click Here

Click Here

Click Here

To purchase at us Click Here

www.CampusRecruitment.co.in
For Details, contact: sales@campusrecruitment.co.in
Or call us at: 07032206275
RATIO PROPORTION
CONCEPTS 1 1
5) Inverse or reciprocal ratio of a : b= :
Ratio: A ratio is the relation between two quantities a b
2
which is expressed by a fraction. b
6) Third proportional to a and b is
The ratio of the number 'a' to the number 'b' is a
a 7) If a : b = x : y and b : c = p : q, then
written as (or) a : b or a to b
b xp
a) a : c=
e.g.: The ratio of 5 hours to 3 hours can be written as y q
5 b) a : b : c = px : py : qy
(or) 5 : 3.
3 c e
a
8) Compound Ratio of (a : b), (c : d), (e : f) is .
The ratio is always a comparison between the b d f
quantities of same kind or of same units. 9) The ratio in which two kinds of substances must be
For example, you cannot form the ratio between 5 hours mixed together one at x per kg and another at y per
and 3 days. Because the two numbers are expressed in kg, so that the mixture may cost n per kg. The ratio is
different units. Hence, convert 3 days to hours. n y
i.e. 3 days = 72 hours. Thus the proper form of this ratio .
x n
5 10) Let the incomes of two persons be in the ratio of a : b
is (or) 5 : 72.
72 and their expenditure be in the ratio of x : y and ff the
Two quantities which are being compared (a : b) are savings of each person is n then income of each is
called its terms. The first term (a) is called antecedent and an( y x ) bn (y x )
second term (b) is called consequent. and respectively.
ay bx ay bx
The ratio of two quantities is always an abstract 11) In a mixture the ratio of milk and water is a : b. In
number (without any units). this mixture another n liters of water is added, then the
If the terms of a ratio are multiplied or divided by the ratio of milk and water in the resulting mixture became
same quantity the value of the ratio remains unaltered. a : m. Then, the quantity of milk in the original mixture
e.g.: The ratio a : b is same as Ma : Mb. an
Proportion: Equality of two ratios is called proportion. = and the quantity of water in the original
m b
Consider the two ratios, a : b and c : d, then proportion
bn
is written as, mixture =
m b
a c
a : b :: c : d (or) a : b = c : d (or) = 12) In a mixture of n liters, the ratio of milk and water is
b d
x : y. If another m liters of water is added to the mixture,
Here a, b, c, d are called Terms. a, d are called Extremes
the ratio of milk and water in the resulting mixture =
(end terms) and b, c are called Means (middle terms).
xn :( yn+mx+my)
4
e.g.: Since the ratio 4 : 20 (or) is equal to the ratio 13) If four numbers a, b, c and d are given then
20 ad bc
1 a) should be added to each of these
1 : 5 (or) we may write the proportion as 4 : 20 :: 1 : 5 (b+c) ( a+d )
5 numbers so that the resulting numbers may be
4 1 proportional.
or 4 : 20 = 1 : 5 or =
20 5 ad bc
In a proportion, product of means (middle terms) is b) should be subtracted from each of
(a+d) (b+c)
equal to product of extremes (end terms). these numbers so that the resulting numbers may be
a c proportional.
i.e. ad= bc or = .
b d
Key Notes: If a and b are two quantities, then
1) Duplicate ratio of a : b=a 2 : b2
2) Sub-duplicate ratio of a : b= a: b
3) Triplicate ratio of a : b=a 3 : b3
4) Sub-triplicate ratio a : b= a: b
3 3

Log on to www.CampusRecruitment.co.in and ask doubt with Question Id. 1.19


Find the details below to purchase the book.

How to order
Click Here

Click Here

Click Here

To purchase at us Click Here

www.CampusRecruitment.co.in
For Details, contact: sales@campusrecruitment.co.in
Or call us at: 07032206275
294 8 23) 8 men : 11 days
L and M complete the work in = 22 days
13 13 11 men : x
Ask doubt with Question Id: 4988 If number of men increase, number of days will decrease.
18) I strike to II strike = 1 interval Hence, arrows are in opposite direction.
II strike to III strike = 1 interval 8 x 118
= x= =8 days
i.e. 2 strikes has one interval time 11 11 11
3 strikes has 2 interval time Ask doubt with Question Id: 7676
10 strikes has 9 interval time 24) 2 kg = 2000 gms.
Similarly, 6 strikes time = 5 interval time 2000 gm : 80 rupees
9 intervals time = 27 seconds 750 gm : x
5 As weight decreases, price decreases.
5 intervals time = 27=15 sec
9 2000 80
= x= 30
Ask doubt with Question Id: 4989 750 x
19) Ask doubt with Question Id: 7677
P1 P2 P3 P4 P5 25) More shadow, more height.
Direct proportion between height and shadow.
d d d d 12.5 28.75
Poles = 5; Distance = 4d = x=17.5
x 40.25
Similarly, 18 poles has 17 intervals of distance Ask doubt with Question Id: 7678
i.e. 18 poles has 17d distance 26) Out of 4 parts, 3 parts of the work is completed in 6
Similarly, 25 poles has 24d distance. 6
If car crosses 17d distance in 34 minutes then it crosses days. For 1 part it takes = 2 days.
3
24 d
'24d' distance in = 34=48 min. Hence, to complete the last part it takes 2 more days.
17 d Ask doubt with Question Id: 7679
Ask doubt with Question Id: 4990 27) 60 women meal = 100 children meal.
th
1 75 children have taken the meal.
20) P can complete of work in 3 days.
5 75 children = 45 women.
i.e. P can do the complete work in 3 5 = 15 days. Still food is left for 25 children.
Q can do the complete work in 4 4 = 16 days. i.e. 25 children = 15 women.
1 1 31 Ask doubt with Question Id: 7680
One days work of P and Q = + =
15 16 240 28) 6 m : 22.8 kg
240 23 11.25 m : x
P and Q can complete the work in = =7 days
31 31 As length increases, weight also increases.
Ask doubt with Question Id: 4991 6 22.8
= x=42.75 kg
21) 4 machines produce 140 bottles/min 11.25 x
140 Ask doubt with Question Id: 7681
1 machine produce = =35 bottles/min
4 29) Men Work Days
1 machine/minute = 35 bottles 26 : 13 : 13
9 machine/ minute = 359 = 315 bottles 39 : 13 : x
Now, for 4 minutes, 354 = 1260 bottles 26 13 x 2613
= x= = 8.66 more days.
Ask doubt with Question Id: 7674 39 13 13 39
22) Pumps Days Cannot be completed in 7 days.
12 15 Ask doubt using Question Id: 7682
9 x 30) Let required number of days be x.
As number of days decrease, number of pumps increase. 5 hens eat 4 packets in 20 days.
15 x 20
= 9 x=1512 x=20 pumps 1 packet= =5 days (5 hens in 5 days eat 1 packet)
9 12 4
Extra pumps = 2012 = 8. Then 1 hen requires 55 = 25 days.
Ask doubt with Question Id: 7675 Ask doubt using Question Id: 7683
Log on to www.CampusRecruitment.co.in and ask doubt with Question Id. 1.32
PERCENTAGES
CONCEPTS e.g.: If a book costs 80 and few monthes later it was
A percentage is a way of expressing a number as a offered at a price of 64. What was the discount
fraction of 100. The word 'per cent' or 'percentage' percentage on that book?
means for every one hundred. In other words, it gives Explanation: Change = 8064 = 16. Original Value = 80.
rate of a parameter per hundred. It is denoted by the 16 1
Discount Percentage = 100= 100= 20%
symbol %. 80 5
30 Calculating Successive Percentages:
e.g.: 30% means 30 out of one hundred or .
100 If a number is successively increased by x% and y%
Key Notes: then a single equivalent increase in that number will be
To convert a percent into a fraction, divide by 100.

e.g.: 20 %=
20 1
=
(
x+ y+
xy
100
%.
)
100 5 e.g.: The price of an article is successivey increased by
To convert a fraction into a percent, multiply by 100. 10% and 20%. What is the overall percent increase in the
3 3 price of the article.
e.g.: = 100=75 %
4 4 Explanation:
To write a decimal as a percent we move the decimal 10% Increase 20% Increase
point two places to the right and put the % sign. 100 110 132
35 Overall 32% Increase
e.g.: 0.35= =35 %
100 (or) By using formula:

( ) ( )
Conversely to write a percent as a decimal, we drop xy (10)(20)
the % sign and insert or move the decimal point two = x+ y+ %= 10+20+ %=30+ 2=32 .
100 100
places to the left.
If there's an increase and a decrease, in that case, the
e.g.: 43% = 0.43; 12% = 0.12.
decrease will be considered a negative value.
Calculating a Percentage:
e.g.: If there is an increase of 20% and then a decrease of
Percentage=
( )
Value
Total
100. 10% on the price of a commodity, the successive
percentage will be
For example, if you obtained 18 marks out of 25 marks, 20( 10)
what was your percentage of marks? 20+( 10)+ =20 10 2=8 % increase.
100
Explanation: Total marks = 25. Marks obtained = 18. In case of discounts, the value of discount percentages
18 will be considered negative.
Percentage of marks obtained = 100=72 %.
25 e.g.: If a shop keeper give 20% and 10% discounts on a
Calculating Percentage Increase or Decrease: festival day, the final discount given by shopkeeper is
% Increase : ( 50 )( 50)
New value = (1+ Increase %) (Original Value) ( 50)+( 50)+ = -100 + 25 = 75% dicount.
100
% Decrease : If there are three discounts as x%, y% and z% then
New value = (1Decrease %) (Original Value) first find the total discount of x% and y% and using it
e.g.: If a book costs 80 and few months later it was find the total discount with z%.
offered at a 30% discount. How much does the book If the price of commodity increases by x%, the
cost now? percentage should a family reduce its consumption so
Explanation: as not to increase the expenditure on the comodity =
New Amount =
( 30
1
100 )
80=0.7080= 56 x
100+x
100.

Calculating Percent Change: If the price of commodity decreases by x%, the


Percentage change refers to the relative percent change percentage should a family increase its consumption so
of an increase or decrease in the original amount. as not to decrease the expenditure on the comodity =
Change x
Percent= 100 100.
OriginalValue 100 x
Log on to www.CampusRecruitment.co.in and ask doubt with Question Id. 1.33
Find the details below to purchase the book.

How to order
Click Here

Click Here

Click Here

To purchase at us Click Here

www.CampusRecruitment.co.in
For Details, contact: sales@campusrecruitment.co.in
Or call us at: 07032206275
TIME AND WORK
CONCEPTS CONCEPTUAL EXAMPLES
1) If a person completes a piece of work in 'n' days, then 1) A is twice as good a workman as B and takes 10 days
th
1 less to do a piece of work than B takes. Find the time in
work done by that person in one day = part of the
n which B alone can complete the work.
work. a) 22 days b) 25 days c) 23 days d) 20 days
1
th Explanation: Let B alone takes 'x' days to complete the
2) If a person completes part of the work in one work. A is twice as good workman as B.
n
day, then the person will take 'n' days to complete the x
It means A takes days to complete the work.
work. 2
3) The total work to be done is usually considered as x
From the given information we can write x =10
one unit. 2
4) If M1 persons can do W1 work in D1 days and 2x x x
=10 =10 x=20 .
M2 persons can do W2 work in D2 days then 2 2
M1 D1 W2 = M2 D2 W1. Alternate Method: Using formula.
5) If M1 persons can do W1 work in D1 days working T1 Here, k = 2 and n = 10
hours per day and M2 persons can do W2 work in D2 Time taken by B working alone to complete the work=
days working T2 hours per day then kn 210
days =20 days .
M1 D1 T1 W2 = M2 D2 T2 W1. k 1 2 1
6) If A can do a piece of work in 'x' days and B can do it Ask doubt with Question Id: 1179
in 'y' days then A and B working together will do the 2) 25 men can reap a field in 20 days. When should 15
xy men leave the work, if the whole field is to be reaped in
same work in days.
(x+ y ) 37 days after they leave the work.
7) If A, B and C can do a piece of work in x, y and z days a) 5 days b) 4 days c) 3 days d) 4 days
respectively then all of them working together can Explanation: 25 men can reap the field in 20 days.
xyz 1 man can reap that field in 2520 i.e. 500 days.
finish the work in days .
(xy+ yz+ zx) Let 15 men leave the work after x days so that
8) If A is trice as good a workman as B then, remaining 10 men can complete the work in 37 days.
Ratio of work done by A and B = 3 : 1. It means 25 men have worked for x days and 10 men
Ratio of times taken by A and B to finish a work =1:3. have worked for 37 days.
9) If A is 'k' times efficient than B and is therefore able 1
25 x+1037 = 50025 x=500 375=125(or) x= 5
to finish a work in 'n' days less than B, then 2
a) A and B working together can finish the work in 15 men must leave the work after 5 days.
kn Ask doubt with Question Id: 1180
days .
2
k 1 3) A man is paid 30 for each day he works, and forfeits
n 5 for each day he is idle. At the end of 60 days he gets
b) A working alone can finish the work in days. 50. Then, he was idle for ____ days.
k 1
kn a) 20 b) 25 c) 30 d) 50
c) B working alone can finish the work in days. Explanation: Suppose, the man was idle for x days.
k 1
30(60 x) 5x = 50 x = 50
10) If A, working alone takes a days more than A and B
Ask doubt with Question Id: 1181
working together. B alone takes b days more than A and
4) 12 men or 15 women can do a work in 20 days. In
B working together. Then the number of days taken by A
how many days 7 men and 5 women would complete
and B working together to finish the job is ab .
the work?
a) 21.8 b) 22.8 c) 25.3 d) 29
Explanation: or means either men or women.
and means both men and women.
12 men or 15 women 12 men = 15 women
4 men = 5 women.
Log on to www.CampusRecruitment.co.in and ask doubt with Question Id. 1.43
Find the details below to purchase the book.

How to order
Click Here

Click Here

Click Here

To purchase at us Click Here

www.CampusRecruitment.co.in
For Details, contact: sales@campusrecruitment.co.in
Or call us at: 07032206275
12) Let, he bought the mobile phone at x. 18) He sells 0.9 mt pipe at rate of 1 mt pipe.
1 1
th
SP of 0.9 mt = CP of 1 mt
Then x x=7500 (By losing on buying cost)
6 6 Let, CP of 1 mt = 100
5 If SP of 0.9 mt = 100,
x=7500 x = 9000. 1100
6 then SP of 1 mt = =111.11
Ask doubt with Question Id: 7731 0.9
13) For 30000, the man loses 25%. 111.11 100
Profit = 100=11.11%
25 100
x x=30000 x = 40000.
100 Alternate Method: Using direct formula.
Now, the man wants gain of 25%. 100 meters 100

40000
( )
125
100
= 50000
90 meters
100 100 10090
100
100
Ask doubt with Question Id: 7730 100 = = 11.11%
14) Let the price of the article is x. 10090 9
8 108 Ask doubt with Question Id: 7736
A sold to B at 8% Profit = x + x= x 19) Let, SP = 100 ; Then, loss = 20
100 100
CP = (100 + 20) = 120
108 112
B sold to C at 12% Profit = x 20
100 100 Loss % = 100%=16.66%
120
108 108 112
Ratio of the selling prices = x : x Ask doubt with Question Id: 7737
100 100 100
28
= 1: = 25 : 28.
25
Ask doubt with Question Id: 7732
15) Difference between selling prices = 3
In the above explanation, ratio of selling prices = 25 : 28.
The difference of these two (25 and 28) is also 3.
So, one of the selling prices can be either 25 or 28.
Option-d is correct choice.
Ask doubt with Question Id: 7733
16) Checking from options.
Calculating profit percentages.

Option-(a): Profit percentage =


( 5
50 )
100=10 %

Option-(b): Profit percentage =


( 3
20 )
100=15 %

Option-(c): Profit percentage =


(60
6
)
100=10 %

Option-(d): Profit percentage =


(
40
5
)
100=12.5 %

Option-(b) is best, as percentage is highest.


Ask doubt with Question Id: 7734
x 4
17) Let, SP = x and CP = y; 6 x=8 y =
y 3

Gain % =
x y
y
100 =
( )
4
3
1 100 =
100
3
=33.33

Ask doubt with Question Id: 7735

Log on to www.CampusRecruitment.co.in and ask doubt with Question Id. 1.70


INTERESTS AND DISCOUNTS
CONCEPTS 3) If a sum of money at simple interest becomes n times
The money borrowed or lent out for a certain period of itself in T years then in how many years it will
is called the principal or the sum. become m times of itself.
Interest is the money paid for the use of borrowed (m 1 )T
Required time = years
money i.e. extra money paid for using others money is ( n 1)
called interest. 1
4) If simple interest on a sum of money is th of the
Sum of interest and principal is called amount. x
Amount = Principal + Interest principal and the time T is equal to the rate percent R


Simple interest: For a certain period, if the interest on 1
a certain sum borrowed is reckoned uniformly, then it is then Rate = Time 100 .
x
called simple interest. Denoted by S.I= P R T 5) A certain sum is at simple interest at a certain rate for
100 T years. If it had been put at R1% higher rate, then it
Amount = Principal + Simple Interest would fetch x more.
P = Principal; SI = Simple Interest x100
T = Time (in years) R = Rate percent per annum Then the Principal = .
TR 1
Time must be expressed in the same units used for
6) The annual payment that will discharge a debt of P
time in the Rate.
due in T years at the rate of interest R% per annum is
e.g.: If 1000 is borrowed for 3 years at 10% simple
100 P
interest, what is the total amount after 3 yrs .
Explanation: Annual payment RT (T 1)
100T+
Year Principal Interest (10%) Amount 2
1st 1000 100 1100 7) Let the rate of interest for first t1 years is r1% per
2nd
1100 100 1200 annum. r2% per annum for next t2 years and r3% for the
3rd 1200 100 1300 period beyond that. Suppose all together the simple
PRT 1000103 interest for t3 years is 'SI'
(or) S.I= = = 300. 100SI
100 100 Then Principal = .
Amount = Principal + Interest = 1000 + 300 = 1300 t 1 r1 +t 2 r 2+(t 3 t 1 t 2 )r 3
e.g.: If 1500 is invested at 15% simple annual interest, 8) The simple interest on a certain sum of money at r1%
how much interest is earned after 9 months? per annum for t1 years= m. The interest on the same
Explanation: Here time is in terms of months but sum for t2 years at r2% per annum = n. Then the sum =
interest is in terms of years. So, Time must be expressed (m n)100
.
in the same units used for time in the Rate. r 1 t1 r 2 t2
9 Compound Interest: If interest as it becomes due and
i.e. 9 months = years.
12 is not paid to the lender but is added on to the principal,
1500159 then the money is said to be lent at compound interest.
Now, S.I = = 168.75
12100 And the total sum owed after a given time is called the
Key Notes on Simple Interest amount at compound interest for that time.

[( ) ]
T
1) If a sum of money at simple interest amounts to A1 R
in T1 years and A2 in T2 years, then Compound Interest = P 1+ 1
100
A1 T 2 A 2 T 1 (A 2 A1 )100

( )
TC
Principal= Rate= R
T 2 T1 A 1 T 2 A 2 T1 Amount = P 1+
C100
2) A sum of money becomes n times of itself in T years
at simple interest, then the rate of interest is, Where T = Number of years and C = Number of times
100(n 1) compounded annually.
Rate = %
T

Log on to www.CampusRecruitment.co.in and ask doubt with Question Id. 1.71


Find the details below to purchase the book.

How to order
Click Here

Click Here

Click Here

To purchase at us Click Here

www.CampusRecruitment.co.in
For Details, contact: sales@campusrecruitment.co.in
Or call us at: 07032206275
EXPLANATIONS 13) Check with options.
x
2
y
2
Option-a: 52 + 72 = 74 202;
1) x2 y2 = 1160 x + y = 116 x y= =10 Option-b: 72 +92 = 130 202
x +y
Ask doubt with Question Id: 1497 Option-c: 92+112 = 202 = 202;
2) Let the numbers be 3x, 4x Hence, optionc is correct.
(3x + 4x) = 420 (or) x = 420 7 = 60 Ask doubt with Question Id::7860
Smaller number = 3x = 180 14) By given condition, let the other number is x.
Ask doubt with Question Id: 1498 6+x = 5(6x) (or) x+6 = 5(x6)
3) Let the other number be x. There can be 2 possibilities 6+x = 305x (or) x+6 = 5x30
x+10 = 2(x10) (or) x+10 = 2(10x) x = 30 (or) 3.33 6x = 24 (or) 4x = 36 x = 4 (or) x = 9
Ask doubt with Question Id: 1499 Hence, both 4 and 9 are correct.
4) Let, first number = x. Then other number = 24 x. Ask doubt with Question Id:7861
7x + 5(24x) = 146 2x = 146120 x = 13. 15) Check with options.
Ask doubt with Question Id: 1500 optiona: 14 + 41 = 55 (optiona is correct)
5) Let the number be x y optionb: 24+42 = 66 55
x + y = 9 and (10x+y)+27 = 10y+x x = 3 and y = 6 Hence, only optiona is correct.
Hence, 36 is the number. Ask doubt with Question Id:7862
Ask doubt with Question Id: 1501 16) Let the number be 3x and x.
6) (10x + y) (10y + x) = 9(x y) 3x + x = 28 4x = 28 x = 7
This is always divisible by 9. Largest number = 3 x=37=21.
Ask doubt with Question Id: 1502 Ask doubt with Question Id:7863
3 17) Let 5 consecutive natural numbers are:
7) 10y + x = of (10x + y) and x + y = 9 x, x+1, x+2, x+3, x+4.
8
(x)+( x+1)+(x +2)+(x +3)+(x +4)
These equations when solved for x and y will give =4
x = 7, y = 2. 5
Ask doubt with Question Id: 1503 5x +10 = 20 5x = 10 x = 2
8) If the number is (xy), then y =4x and 10x+y+27 = 10y+x Smallest number= x = 2
Solving these equations, we get, x = 1, y = 4. Ask doubt with Question Id:7864
Thus 14 is the required number. Its square is 196. 18) Let, the number be x.
Ask doubt with Question Id: 1504 By given condition, x + x= 400 x + x=20
x +2 3 Now, by checking with options:
9) = 5 x+10=66 3 x x=7 optiona: 4 + 4 = 4+2 = 6 20
20 x+2 5
7 optionb: 16+ 16 = 16+4 = 20
Hence the fraction is Option-b is correct.
13
Ask doubt with Question Id: 1505 Ask doubt with Question Id: 7865
x 19) Let, Salman has x then Sohail has x2.
10) If be the fraction. x+x2 =110 x2+x110 =0 x2+11x10x110=0
y
x(x+11)10(x+11)=0 x = 10, x = 11
x+1 1 x 1
then = and = x = 3 and y = 8. Rupee is positive, so, x = 10.
y 2 y+1 3
Sohail has x2 = 102 = 100.
Ask doubt with Question Id: 1506
Ask doubt with Question Id: 7866
11) 10x + y = 4 (x + y) and 10x + y + 18 = 10y + x
20) Let, the digits are x and y. 10x + y + 9 = 10y + x
x = 2, y = 4. The two digit number = 24.
9x 9y = 9 x y = 1 .................... (1)
Ask doubt with Question Id: 1507
By other condition, x + y = 3 .................... (2)
12) x 9=+10
() 1
x
x 9=
+10
x
x2 9x 10 =0 Solving (1) and (2), x = 1, y = 2
Number = 10x + y = 101+2 = 12
(x10) (x+1) = 0 i.e. x = 10, 1. Ask doubt with Question Id: 7867
Number is positive i.e. x = 10
Ask doubt with Question Id: 7859

Log on to www.CampusRecruitment.co.in and ask doubt with Question Id. 1.87


Find the details below to purchase the book.

How to order
Click Here

Click Here

Click Here

To purchase at us Click Here

www.CampusRecruitment.co.in
For Details, contact: sales@campusrecruitment.co.in
Or call us at: 07032206275
10) Centroid: The point of concurrence of the medians A D
of a triangle is called Centroid and is denoted by 'G'.
11) If G is the Centroid of ABC, then
B C E F
a) AD is one of its median then G divides AD in the
AB BC AC
ratio 2 : 1. i.e. ABC ~ DEF = =
DE BF DF
b) AB 2 + BC 2 + CA 2 = 3 (AG 2 + BG 2 + CG 2 ).
A.A.A. Similarity: If the corresponding angles of two
B
triangles are equal then they are similar.
S.S.S. Similarity: If the ratio of the corresponding sides
G
of two triangles are equal then they are similar.
C A S.A.S Similarity: If the ratio of two corresponding sides
1 of two triangles are equal and their included angles are
c) ABG = BCG = ACG = ABC . equal then they are similar.
3
QUADRILATERALS
B A quadrilateral is a plane figure bounded by four
D G straight lines called sides.
A The sum of the four angles of a quadrilateral is 360o.
C
Types of Quadrilaterals:
12) In a ABC if D, E, F are the mid points of the sides
A trapezium is a quadrilateral with one pair of
BC, CA and AB respectively then
parallel opposite sides.
1
AEF = BDF = CDE = DEF = ABC and Perimeter = Sum of all sides a
4
( a+b)h
3(AB2 + BC2 + CA2) = 4(AD2 + BE2 + CF2). Area = h
2
A

E b
F
A parallelogram is a quadrilateral with
B C (a) two pairs of parallel opposite sides
D (b) two pairs of equal oppsite sides
13) If the lengths of 2 medians of a triangle are equal (c) two pairs equal opposite angles.
then it is an isosceles triangle. If the lengths of 3 Perimeter = 2(a+b) yo
medians of triangle are equal then it is an equilateral xo
Area = a h.
triangle. h
xo yo b
14) In an equilateral triangle with side 'a' and altitude

'h' we have 'h' =


3
a and Area=
3 2
a or
h
2

. a
2 4 3 A rhombus is a parallelogram with
1 (a) four equal sides
15) The area of a ABC = base height .
2 (b) equal opposite angles a yo a
d1
1 1 1 (c) no parallel sides. xo xo
= a b sin C = b c sin A = a c sin B
2 2 2 Perimeter = 4a d 2
16) If in a ABC, a circle is inscribed by touching the 1 yo
Area = d 1 d 2 a a
sides at P, Q, R respectively then 2
AP+BQ+CR = PB+QC+RA = (AB+BC+CA) In a rhombus, d12 + d22 = 4a2.
B Here, d1 and d2 are two diagonals.
R A square is a rhombus with
P (a) 2 pairs of parallel lines
(b) 4 equal sides
C A
Q (c) 4 equal internal right angles.
Similar triangles: Two triangles ABC, DEF are said to Perimeter = 4a. 90o
be similar if their corresponding angles are equal (or) Area = a2. Diagonal = 2 . a a
the ratio of their corresponding sides are equal.
Log on to www.CampusRecruitment.co.in and ask doubt with Question Id. 1.90
Find the details below to purchase the book.

How to order
Click Here

Click Here

Click Here

To purchase at us Click Here

www.CampusRecruitment.co.in
For Details, contact: sales@campusrecruitment.co.in
Or call us at: 07032206275
PERMUTATIONS AND COMBINATIONS
CONCEPTS The number of permutations of 'n' objects taken 'r' at a
Fundamental Principal of Multiplication: time will be denoted by P(n, r).
In general if some procedure can be performed in n1 Before we derive the general formula for P(n, r) we
different ways, and if, following this procedure, a consider a special case. Find the number of
second procedure can be performed in n2 different permutations of 7 objects, say a, b, c, d, e, f, g taken three
ways, and if, following this second procedure, a third at a time. In other words, find the number of 'three letter
procedure can be performed in n3 different ways, and so words' with distinct letters that can be formed from the
fourth then the number of ways the procedure can be above seven letters.
performed in the order indicated is the product Let the general three letters word be represented by
n1 . n2 . n3. ........ three boxes.
e.g.: A letter lock consists of 5 rings each marked with 10
different letters. What is the maximum number of
unsuccessful attempts to open the lock. Now the first letter can be chosen in 7 different ways;
Explanation: Each ring is marked with 10 different following this, the second letter can be chosen in 6
letters. Hence each ring has 10 positions. different ways; and, the last letter can be chosen in 5
Thus, the total number of attempts that can be made to different ways. Write each number in its appropriate
open the lock is 10 x 10 10 10 10 = 105 box as follows:
Out of these, there must be one attempt in which the
lock will open. 7 6 5
Total number of unsuccessful attempts = 105 -1
Thus by the fundamental principle of counting there are
Fundamental Principle of Addition:
7.6.5=210 possible three letter words without repetitions
If there are two operations such that they can be
from the seven letters. (or) There are 210 permutations
performed independently in m and n ways respectively,
of 7 objects taken 3 at a time.
then either of the two operations can be performed in
i.e. P(7, 3) = 210.
(m+n) ways
The derivation of the formula for P(n, r) follows the
Factorial: The product of first 'n' natural numbers is
procedure in the preceding example:
called the 'n'-factorial and is denoted by n!
The first element in an r-permutation of n-objects can be
n! = 1.2.3.4 . ................ (n2).(n1).n
chosen in 'n' different ways; following this, the second
Example: 4! = 1.2.3.4 = 24, 5! = 1.2.3.4.5 = 125,
element in the permutation can be chosen in
5! = 5.4! = 5.24 = 120, 6! = 6.5! = 6.120 = 720.
(n1) ways; and, the third element in the permutation
Note: 1) 0!=1
can be chosen in (n2) ways. Continuing in this manner,
2) The product of 'r' consecutive positive integers is
we have that the rth (last) element in the
divisible by r!
rpermutation can be chosen in n(r1) = nr+1 ways.
3) (kn)! Is divisible by (n!)k for all k is a positive
n!
constant. Thus P(n, r) = n(n1) (n2) . . . (nr+1) =
(n r)!
4) The product of 2n! consecutive positive integers is
The second part of the formula follows from the fact
equal to 2(n!).
that n(n1)(n2) . . . . . . .(nr+1) =
PERMUTATIONS
n (n 1)( n 2 )(n r+1)( n r )! n!
Permutation: An arrangement of any r n of these =
objects in a given order is called an rpermutation or a (n r !) (n r)!
permutation of the 'n' objects taken 'r' at a time. A formula for the number of possible permutations of
Example: Consider the set of letters a, b, c, and d. Then n!
'r' objects from a set of 'n' is P(n, r) or n p r =
(i) bdca, dcba and acdb are permutations of the 4 letters (n r )!
taken all at time.
(ii) bad, adb, cbd and bca are permutations of the 4 letters In the special case that r = n, we have P(n, n) = n(n1)
taken 3 at a time. (n2). . . . . . 3.2.1 = n! (in other words there are n!
(iii) ad, cb, da and bd are permutations of the 4 letters permutations of 'n' objects taken all at a time).
taken 2 at a time.

Log on to www.CampusRecruitment.co.in and ask doubt with Question Id. 1.110


Find the details below to purchase the book.

How to order
Click Here

Click Here

Click Here

To purchase at us Click Here

www.CampusRecruitment.co.in
For Details, contact: sales@campusrecruitment.co.in
Or call us at: 07032206275
PROBABILITY
CONCEPTS Events: An event A is a set of outcomes or, in other
Random Experiment: Probability is the study of words, a subset of the sample space S.
random or non deterministic experiments. If the die is Example: If A random experiment is associated with
tossed in the air, then it is certain than the die will come what is the day today. It may be from Sunday to
down, but is non certain that, say a 3 will appear. Saturday. If today is Friday and Friday belongs to the
Definition: A random experiment is an experiment whose sample space S = {Sun, Mon, Tue, Wed, Thu, Fri, Sat}
result would not be predicted but the list of possible Different Types of Events:
outcomes are known. The unpredicted outcomes could Simple or Elementary Events: An event with only
not be taken under random experiments. The result of one sample point is called simple or elementary event.
random experiments may not be predicted exactly but In a experiment of tossing three coins at a time, the
the result must be with in the list of predicted outputs. event 'A' is that all coins turns up with heads consists of
Example: only one point HHH. Then 'A' is a simple event.
1) Tossing a fair coin. As a matter of fact each outcome of an experiment is a
2) Rolling a die is a random experiment, since its results simple event.
could not be predicted in any trial. Complimentary Event: An event A (or A1) is said to
3) Selection of a plastic component and verification of its be complementary to an event 'A' in sample space 'S'
compliance. consists of all those points which are not in 'A'.
4) Life time of a computer. Example: In tossing a coin three times, sample space S
5) Number of calls to a communication system during a consists of eight points.
fixed length interval of time. S = {HHH, HHT, HTH, THH, HTT, TTH, THT, TTT}
Outcome: The result of a random experiment will be The event 'A' is such that there should be no heads in
called an outcome. the sample point is {TTT}. Then the event A (or A1)
Example: complementary to 'A' is that there exists at least one
1) Tossing a coin. The result is ether Head(H) or Tail(T) head in the sample space i.e. (HHH), (HHT), (HTH),
2) In an experiment of throwing a six-faced die. The (THH), (HTT), (TTH), (THT).
possible outcomes are 1, 2, 3, 4, 5 and 6. Equal Events: Two events A and B are said to be equal
Sample space: The set of all possible outcomes of if AB and BA. This statement implies that all the
some given experiment is called sample space. A points of A are also the points of B and vice-versa.
particular outcome, i.e. an element in in that set is called Example: Let sample space S = {1, 2, 3, 4, 5, 6, 7, 8, 9, 10}.
a sample point or sample. Let A be the event defined as 'even number' and event B
Example: is defined as 'multiplies of 2'.
1) Toss a die and observe the number that appears on Then A = {2, 4, 6, 8, 10}; B = {2, 4, 6, 8, 10} here every
top. Then the sample space consists of the six possible point in A is also a point in B and vice-versa. There fore
numbers: S = {1, 2, 3, 4, 5, 6} events A and B are said to be equal events.
2) Toss a coin 2 times and observe the sequence of heads Transitivity of events: If A, B and C are 3 events such
(H) and tails (T) that appears. Then the sample space S that AB and BC it implies that AC such a property
consists of four elements: S = {HH, HT, TH, TT} of events is known s transitivity of events.
Note: Shortcut: Tossing a coin 2 times is same as tossing Let the sample space S= {1, 2,...., 100}.
2 coins at a time. Event A be the 'even numbers': A = {2, 4, 6, 8,.....,100}
S = {H, T} {H, T} Event B be the 'multiples of 4' : B = {4, 8, 12,.......,100}
S = {HH, HT, TH, TT} Event C be the 'multiples of 8': C = {8, 16, 24,.....,100}
3) Toss a coin until a head appears and then count the Event point in C is also point in B and event point in B is
number of times the coin was tossed. The sample space also point in A but not viceversa i.e. ABC.
of this experiment is S = {1, 2, 3, ... }. Here refers to Compound event: An event which is not simple or
the case when a head never appears and so the coin is elementary is called a compound event. Every compound
tossed an infinite number of times. This is an example of event can be uniquely represented by the union of a set
a sample space which is countably infinite. of elementary events.

Log on to www.CampusRecruitment.co.in and ask doubt with Question Id. 1.118


Find the details below to purchase the book.

How to order
Click Here

Click Here

Click Here

To purchase at us Click Here

www.CampusRecruitment.co.in
For Details, contact: sales@campusrecruitment.co.in
Or call us at: 07032206275
DATA SUFFICIENCY
CONCEPTS 3) What is the value of x?
Purpose of Data sufficiency: I. x2+2x 3=0 II. x2+4x 5=0
Hear the examiners intention is to check the student's Explanation: From statement-I, x2+2x-3=0
capability in decision making. One can agree that the x2+3x-x-3 = 0 x(x+3)-1(x+3)=0 x = 1 or 3
decision making is the sense of checking whether the From statement-I alone we can't say exact value of x.
data is sufficient or not. From statement-II, x2+4x-5=0 x2+5x-x-5=0
Nature of questions: You will be given a question x(x+5)-1(x+5)=0 i.e. x=1 or -5.
followed with the two statements. From statement-I and II, we conclude, x = 1.
You dont need to solve the question. You just have to As both the statements together are required to answer
judge whether given two statements have enough the given question, option-e is correct.
information to solve the question. Ask doubt with Question Id: 5505
CONCEPTUAL EXAMPLES 4) Find the area of the square?
Each of the questions below consist of a question and I. The side of the square is 7 cm
two statements numbered I and II. You have to decide II. The circumference of the square is 28 cm
whether the data provided in the statements are Explanation: Area = (side)2
sufficient to answer the given question. Read both the From statement-I, we know the value of side. Therefore
statements and give answer as area can be found.
a: If the data in statement-I alone is sufficient and the From statement-II, circumference i.e. 4(side)=28.
data in statement-II alone is not sufficient to answer the From this we can find the value of side. As a result area
question. can also be found.
b: If the data in statement-II alone is sufficient and the Here, either of the statements-I or II alone are sufficient
data in statement-I alone is not sufficient to answer the to answer the given question. Hence, option-c is correct.
question. Ask doubt with Question Id: 5506
c: If the data either in statement-I or in statement-II 5) What is the cost price of the chair?
alone are sufficient to answer the question. I. The selling price of the chair is 324 at profit of 8%.
d: If the data either in statement-I and II together are not II. The profit is 12%.
sufficient to answer the questions. And some more data Explanation:
needed. 100
From statement-I, CP= 324= 300
e: If the data in both statement-I and II together are 100+8
necessary to answer the question. Statement-I alone is sufficient to answer.
1) What is the average of p, q and r? Statement-II does not have the enough information to
I. r is 25. II. p + q is 20. solve the given question. Hence, option-a is correct.
Explanation: To find the average, we need values of p, Ask doubt with Question Id: 5507
q, r. From the given two statements values of p, q, r are 6) Who is tallest?
known. Hence, we require both the statement-I and II to I. C is eldest.
answer the given question. Hence, option-e is correct. II. A is shortest and B is youngest but taller than C.
Ask doubt with Question Id: 5503 Explanation:
2) Who is youngest among Raju, Vamsi and Rajni? Statement-I alone is not sufficient to answer.
I. Raju is one year elder to Vamsi. From statement-II, A is shortest. And B is taller than C.
II. Vamsi age is average age of Raju and Rajni. It means B is taller than A and C. i.e. only statement -II is
Explanation: sufficient to answer the question. Hence, option-b is
From statement-II, Vamsi's age is between the ages of correct.
Raju and Rajni. Ask doubt with Question Id: 5508
From statement-I, Raju is one year elder to Vamsi. It 7) Is r > s ?
means Rajni will be one year younger to Vamsi. I. r > t II. at > ar, a < 0.
From both the statements, we can say, Rajni is Explanation: Statement-I and II gave information about
youngest among the three. Hence, option-e is correct. t and r. But not s. So, it is not possible to say whether
Ask doubt with Question Id: 5504 r > s or not because of insufficient information from both

Log on to www.CampusRecruitment.co.in and ask doubt with Question Id. 1.129


Find the details below to purchase the book.

How to order
Click Here

Click Here

Click Here

To purchase at us Click Here

www.CampusRecruitment.co.in
For Details, contact: sales@campusrecruitment.co.in
Or call us at: 07032206275
DATA INTERPRETATION
CONCEPTS 1) Give the total percentage of Girls who wrote SSC
The information related to any event given in the form examination from that School.
of graphs, tables, charts etc is termed as data. The a) 25% b) 54% c) 23% d) 58%
methodology of interpreting data to get the 2) Give the percentage of students who scored
information is known as data interpretation. distinction (> 75).
Mathematical identities which we use in data a) 43% b) 34.25% c) 24.85% d) 40%
interpretation are given below. 3) Give fail percentage of students in SSC examination.
To solve the problems on data interpretation, you need a) 1% b) 2% c) 4% d) 8%
to be thorough in 'Percentages', 'Ratios' and 'Averages' 4) Give pass percentage of boys in SSC examination.
chapters. a) 90% b) 88% c) 98% d) 99%
Percentage: Proportions with the base 100 are known 5) Give the percentage of students who scored more
as percentages (%). than 60% in the SSC examination.
x a) 25% b) 59.3% c) 22.2% d) 50%
For example, 100 is in percentage form.
y Explanation:
e.g.: If the ratio of boys to total number of students in a 1)b; Total no.of girls appeared for SSC Examination =
1015 25 + 15 + 10 + 5 + 3 = 58.
college is . This can be written in a percentage Total no.of students appeared for SSC examination =
4060
1015 58 + 50 = 108.
form as 100=25 %. Percentage of girls who wrote SSC Examination =
4060
58
To find by how much percent 'x' is more or less than y 100=53.7=54 % (apporoximately)
(or over y) when compared to y is given as 108
Value of X Value of Y 2)b; No.of students who scored distinction = 25 +12 =37.
Required Percentage= 100 Percentage of students who scored distinction =
Valueof Y
37
Observe that the denominator contains the value with 100=34.25%
which the comparison is made. 108
In the above formula, if numerator is positive, then 3)c; Total no.of students failed in SSC examination = 4.
4
there is percentage growth. If numerator is negative, Fail % = 100= 3.7= 4 % (apporoximately)
then there is a decline in the percentage. 108
Ratio: In the simplest possible form, ratio is a quotient 4)c; No.of boys passed in the examination = 49.
49
or the numerical quantity obtained by dividing one Boys pass percentage = 100=98%
figure by the other figure of same units. 50
For example, in a school, number of 5)b; No.of students who scored more than 60% = 64.
Percentage of students who scored more than 60% =
TABULAR DATA INTERPRETATION
64
In this type of questions a table with data as well as a 100=59.26%=59.3 % (Apporoximately)
set of questions on the same data is given to you. You 108
need to analyze the table data and answer the given Example: Production of cars by different companies
questions. in the span of 1980-2005 given. Interpret the data to
Example: Study the following table carefully and answer the questions given below.
answer the questions that follow. Production of cars in thousands
Company
Table: Percentage of marks scored by students in SSC 1980-85 1985-90 1990-95 1995-00 2000- 05
Marks percentage Girls Boys Maruthi 12.5 15.0 16.2 18.0 22
>75 25 12 Hindustan
10.4 11.1 11.5 11.5 12
60-75 15 12 Motors
50-59 10 23 Hyundai
12 14.3 16.2 17.8 18.9
35-49 5 2 Motors
< 35 3 1 Ford 14.4 14.1 13.2 18.1 25.3
General
19.2 13.8 13.5 14.1 15.8
Motors
Log on to www.CampusRecruitment.co.in and ask doubt with Question Id. 1.135
Find the details below to purchase the book.

How to order
Click Here

Click Here

Click Here

To purchase at us Click Here

www.CampusRecruitment.co.in
For Details, contact: sales@campusrecruitment.co.in
Or call us at: 07032206275
BAR GRAPHS
Bar graphs normally comprise X-axis, Y-axis and bars. 5)d; Average of imports during 2000-2005 =
X and Y-axes represent the data. And bars represent 300+500+600+ 550+ 400 2350
= =470
the trend of data with respect X and Y-axes. In this type 5 5
of questions, data is given in the form of bar graphs. Average of exports during 2000-2005 =
You need to analyze the bars with respect to X and Y- 400+600+500+150+350 2000
= = 400
axes to answer the given questions. 5 5
Example: Imports and exports of a country from 2000 Difference = 470 400 = 70
- 2001 to 2004 -2005. Example: Turnover in crores of six companies (U, V,
700 W, X, Y and Z) are given.
600 9
8
500
7
400 6
300
5
4
200 3
100 2
1
0
2000-2001 2001-2002 2002-2003 2003-2004 2004-2005 0
U V W X Y Z
Imports Exports 1) Which company's turn over is highest?
1) In which of the following year the gap between a) U b) V c) W d) X
import and export was maximum. 2) What is the percentage of turn over of the company-
a) 2001-02 b) 2002-03 c) 2003-04 d) 2004-05 X over the turn over of the company-V?
2) In which of the following year the gap between a) 25% b) 50% c) 75% d) 100%
imports and exports was minimum. 3) Give the difference of average turnovers of first three
a) 2002-2003 b) 2003-2004 c) 2004-2005 d) none companies and last three companies.
3) Exports in 20012002 was approximately how many a) 3.33 b) 6.66 c) 2.48 d) 1.85
times that of the year 20032004. 4) Give the percentage contribution of turnover of W in
a) 2 b) 3 c) 4 d) 5 the overall turnover of all the companies.
4) Give the ratio between the number of years in which a) 12% b) 50% c) 40% d) 29%
exports is greater than imports and import is greater 5) Difference of average percentage contribution of
than exports. turnovers of companies U, V and X, Y is.
a) 3 : 2 b) 2 : 3 c) 3 : 1 d) 1 : 3 a) 1% b) 2% c) 3% d) 4%
5. Difference between average of imports and exports is Explanations:
a) 100 b) 90 c) 80 d) 70 1)c; It is clear from the graph that turn over of company
Explanations: W is highest i.e. 7 crores.
1)c; From the graph, gap between import and export 2)b; Turnover of company X = 3 crores
was maximum in 2003-2004. Turnover of company V = 6 crores
2)c; From the graph, gap between imports and exports Percentage of turn over of X over V =
is minimum in 2004-2005 = 400350= 50 core. 3
100=50 %
3)c; Exports of the year 20012002 = 600 6
Exports of the year 20032004 = 150 3)a; Average turn over of first three companies
Exports of 20012002 is 4 times greater than that of 4+6+8 20
= =6.66
20032004. 3 3
4)b; In 2 years i.e. 2000-2001 and 2001-2002 exports are Average turn over of last three companies
greater than imports. 3+5+2 10
= = 3.33
In 3 years i.e. 2002-2003, 2003-2004, 2004-2005 imports 3 3
are greater than exports. Difference = 6.66 3.33 = 3.33
Log on to www.CampusRecruitment.co.in and ask doubt with Question Id. 1.143
Find the details below to purchase the book.

How to order
Click Here

Click Here

Click Here

To purchase at us Click Here

www.CampusRecruitment.co.in
For Details, contact: sales@campusrecruitment.co.in
Or call us at: 07032206275
MIXED DIAGRAMS
CONCEPTS B) Study the following graphs carefully to answer the
In this type of data interpretation, data will be given in questions given below it. (Use most approximate
the form of two or more diagrams. The combination of figures, if necessary).
the diagrams can be a bar diagram and a pie chart (or) 80
a line graph and a table diagram (or) a pie chart and 70
line graph.
60
A) Study the following graphs carefully to answer the
50
questions given below it.
Readers of news papers in percentages in 3 different 40
cities A, B and C over the years. 30
80% 20
70% 10
60% 0
50% R M Y A G O W R
40% Different Bags containing colored (red, yellow, green,
30%
white) balls
Different Bags containing fruits and Flowers (Mango,
20%
Apple, Orange, Rose)
10%
Percentage of bags (empty, fruits and Flowers,
0%
2001 2002 2003 2004 colored balls) available in every house is given in the
A B C
following Pie chart.

Total population in 3 cities is represented in the


following diagram. Empty bags
3 Crores 200o
A 120o
110o Fruits &
1000
B C
Coloured flower
balls Bags bags
1) In 2002 in the city B how many people were reading
a news paper in lakhs ? There are 3 go-downs namely AB, BC, CE which have
a) 108.333 b) 59.5883 c) 48.7499 d) 38.9421 n bags in different days of the week as shown in the
2) According to the data in city B what is the difference table.
between minimum number of news paper readers in a
particular year and maximum number of newspaper
readers in a particular year (aprx)?
a) 34 b) 31 c) 29 d) 27
3) What is the sum of populations of city A those who
don't read any newspapers in all the 4 years (in lakhs)? 216
a) 220 b) 200 c) 180 d) 160
4) What is the ratio between news paper readers in the
three cities A, B and C in the year in which city B has
maximum readership (aprx)? 72
a) 9 : 10 : 6 b) 6 : 9 : 10 c) 10 : 9 : 6 d) none 36
5) In the 2 years in which same and maximum
sun mon tue wed thu fri sat
percentage of readership is maintained in the cities A
and C. What is the decrease in readership in the city A? House AB House AB

a) 5 lakhs b) 10 lakhs c) 20 lakhs d) 30 lakhs House BC

Log on to www.CampusRecruitment.co.in and ask doubt with Question Id. 1.158


Find the details below to purchase the book.

How to order
Click Here

Click Here

Click Here

To purchase at us Click Here

www.CampusRecruitment.co.in
For Details, contact: sales@campusrecruitment.co.in
Or call us at: 07032206275
LOGICAL REASONING
CODING - DECODING
CONCEPTS 2) ZYXW as coded as ABCD then STUV is coded as.
A code is a system of words, letters or signs which is Explanation: Here each letter is coded with its opposite
used to represent a message in secret form. Coding and letter. i.e. Z A, Y B, X C, W D.
Decoding test is to examine the student's ability to Similarly, S H, T G, U F, V E.
identify the rule interpreted and decode the given 3) bcd is coded as def then true is coded as.
message. Explanation: Here every letter is moved two steps
Approach to solve the questions: forward. i.e. b (+2) d , c (+2) e, d (+2) f .
1. You will be given two messages, one is original Similarly, t v, r t, u w, e g.
message and another one is coded message. So, the answer is 'vtwg'.
2. You have to compare each element of the original 4) hyderabad is coded as ixedszcze then chennai is
message with corresponding element of coded coded as?
message. Thereafter try to identify the rule in which Explanation:
coded message is formed. Here the letters are alternatively increasing and
3. Using the identified rule you can easily answer the decreasing by 1.
question asked. h(+1) i , y(-1) x , d(+1) e , e(-1) d, r(+1) s,
Tips to solve easily: a(-1) z, b(+1) c, a(-1) z, d(+1) e.
1) Remember English alphabets from A to Z with their So, chennai will be coded as dgfmozj.
position values i.e. A1, B2, C3, . . . . . ., Z26. (II) Number Coding: In this type of coding, alphabets
2) Remember reverse order of English alphabets. i.e. Z are assigned to the numbers or numerical code values
to A with their position values i.e. Z1, Y2, .... , A26. are assigned to a word or alphabets. You have to
3) Remember the corresponding opposite letter of each compare the given codes to answer the questions.
alphabet with their position values. The following table Example:
will give the opposite letter of each alphabet. 1) If READ is coded as 7421 and BOOK is coded as
8335, then how would you encode BOARD?
1 2 3 4 5 6 7 8 9 10 11 12 13
Explanation: The alphabets are coded as follows.
A B C D E F G H I J K L M
R E A D B O O K
Z Y X W V U T S R Q P O N
7 4 2 1 8 3 3 5
26 25 24 23 22 21 20 19 18 17 16 15 14 From the above codes, we can say, B is coded as 8, O is
To find out the opposite letter of a particular letter, we coded as 3, A is coded as 2, R is coded as 7, D is coded
can use the below formula. as 1. Hence, BOARD is coded as 83271.
Sum of the position numbers of a letter and it's (III) Substitution: In this type, the names of objects are
opposite letter is always 27. substituted with different names. We should carefully
Position number of a letter + Position number of its trace the substitution to answer the questions.
opposite letter = 27. Example:
e.g.: The opposite letter of 'H' is 'S'. 1) In a certain code language, 'book' is coded as 'pencil',
Because, H8, S19. H+S = 27 8+19=27. 'pencil' is coded as 'mirror', 'mirror' is coded as 'board'.
Types of Coding-Decoding: Then what is useful to write on a paper?
(I) Letter Coding: In this type of coding, the original Explanation: We use pencil to write on a paper but here
alphabets of the given word are replaced by certain pencil is coded as mirror. So, the answer is mirror.
other alphabets based on specific rule to form its code. 2) In a certain language, man' is called as woman,
You have to detect the hidden rule and answer the 'woman' is called as girl, girl is called as boy, boy is
questions accordingly. called as worker. Then in the same language what does
Examples: (1) In a certain code language,'COLLEGE' is a 6 year old female is called?
written as 'GSPPIKI' then how will 'GROUPS' be Explanation: In general language, 6 years old female is
written in that code? called as girl. But in the given coded language girl is
Explanation: Each letter of the word is moved four called as boy. So, the answer is 'boy'.
steps forward to obtain the code. So, GROUPS will be
coded as KVSYTW.
Logon to www.CampusRecruitment.co.in and ask doubt with Question Id. 2.1
(IV) Mixed Letter/ Number Coding: In this type, few 3) In a certain code, DANGER is written as BCLICT. In
sentences are given in a code language. Based on that the same code, ORACLE will be written as?
you are asked to find the code for a particular given a) MTYEJG b) MPYEJG c) MTXCJG
word(s). To answer such questions, you have to d) GJEYTM e) none of these
compare the two messages and deduce the common Explanation: The letters at odd position are moved two
words and its corresponding codes. Analyze the entire steps backward and those at even position are moved
message until the code for the given word is found. two steps forward to obtain the corresponding letters
Example: 1) In a certain code language 'pqost ygx mnlia' of the code. Hence, option-a is correct choice.
is coded as 'manager is there'; 'xyrs kixt ygx' is coded as Ask doubt with Question Id: 7952
'clerk is here'; 'ygx srtv pqost' is coded as 'manager is 4) In a certain code language, SUCCESS is written as
genius'. Then what is the corresponding code for 'there'. 6344866 and NATIVE is written as 279508, then what
Explanation: will be the code for SENSITIVE ?
pqost ygx mnlia manager is there --------- (1) a) 681659508 b) 632659503 c) 683659508
xyrs kixt ygx clerk is here --------- (2) d) 648659504 e) 682659508
ygx srtv pqost manager is genius ------- (3) Explanation: The alphabets are coded as shown below.
From (1) and (2), common word is 'is' and common SUCC ES S NAT I V E
code is 'ygx'. i.e. 'is' 'ygx'.
Similarly, from (1) and (3), manager is coded as pqost. 63 4 4 86 6 2 7 95 0 8
Substitute the codes of 'manager' and 'is' in (1), we get, From the above codes, we can write SENSITIVE as
'there' is coded as 'mnlia'. Hence, mnlia is the answer. 682659508, which is option-e.
2) In a certain code language, '2 4 7' means 'spread red Ask doubt with Question Id: 7953
carpet'; '2 3 6' means 'dust one carpet' and '2 3 4' means 5) In a certain code language, 26891 is written as
'one red carpet'. Then what is the code for the word EGKPT, and 3457 is written as RUAL. Then how 58946
'spread'? is written in that code?
Explanation: 2 4 7 spread red carpet -------(1) a) AKPUG b) GUPKA c) AKUPG
2 3 6 dust one carpet ------- (2) d) AKPGU e) none of these
2 3 4 one red carpet ------- (3) Explanation: Numbers are coded as shown below.
Comparing (1) and (3), we get, 'red carpet' means 2 4. 2 6 8 9 1 3 4 5 7
Hence, 'spread' means 7.
CONCEPTUAL EXAMPLES E GK PT R UA L
58946 is coded as AKPUG; Option-a is correct choice.
1)In a certain code, CONCEPT is written as TQFDOPC.
Ask doubt with Question Id: 7954
Then how VICTORY is written in the same code?
6) In a certain code language blue is called green, green is
a) YROTCIV b) RYOTCIV c) IVCTORY
called yellow, yellow is called white, white is called red,
d) YSPUDJV e) VSPUDJY
red is called purple, purple is called orange. Then what is
Explanation: In the given code, the letters of the word
the color of milk ?
CONCEPT are reversed. After reversing except first
a) White b) Red c) Green d) Orange e) Purple
and last letters, remaining all letters are moved one
Explanation: Actually the color of milk is white. But in
step forward to obtain the code. In the similar manner,
the given language, white is called red. So, answer is
VICTORY is coded as YSPUDJV.
'red'.
Ask doubt with Question Id: 7950
Ask doubt with Question Id: 7955
2) If in a code language, JUMP is written as ITLO and
7) In a certain code language, all the two wheeler
COPIER is written as BNOHDQ, then how will
vehicles are called buses, all the buses are called books, all
CAMPUS be written is that code?
the books are called trees, all the trees are called pens.
a) SUPMAC b) ACPMSU c) BZLOTR
Then what we use to read?
d) DBNQVT e) BLORTZ
a) Books b) Pens c) Trees d) Buses e) none of these
Explanation: Each letter in the word is moved one step
Explanation: We use books to read but in the given
backward to obtain the corresponding code. In the
language books are called trees. So, the answer is trees.
same way, CAMPUS is coded as BZLOTR.
Ask doubt with Question Id: 7956
Ask doubt with Question Id: 7951

Logon to www.CampusRecruitment.co.in and ask doubt with Question Id. 2.2


Find the details below to purchase the book.

How to order
Click Here

Click Here

Click Here

To purchase at us Click Here

www.CampusRecruitment.co.in
For Details, contact: sales@campusrecruitment.co.in
Or call us at: 07032206275
DAY SEQUENCE/ CALENDER
CONCEPTS Hence, in an ordinary year there are 52 perfect weeks
In day sequence, questions will be asked on calendars and 1 odd day. [365 days = 52 weeks + 1 day]
to find a particular day of the week (or) a particular 4) How many odd days are there in a leap year?
day of the given date. In order to solve these problems Explanation: As we know, a leap year has 366 days. So,
easily, you should have knowledge on calendar i.e. leap 7) 366 (52
year, odd days etc. 364
Leap year: If the last two digits of a given year is 2 odd days
perfectly divisible by 4 then that year is a leap year. Hence, in a leap year, there are 52 perfect week and
Example: 2016 is a leap year because last 2 digits i.e. 16 2 odd days. [366 days = 52 weeks + 2 days]
is perfectly divisible by 4. Note: Total number of odd days can be from 0 to 6 only.
But a century year is not a leap year i.e. 100, 200, 300, .... Counting odd days for century years:
But every 4th century year is a leap year i.e. 400, 800, 1) 100 years = 76 ordinary years + 24 leap years.
1200, 1600, 2000 etc. = (761 + 242) odd days = 124 odd days
A leap year has 366 days. (Here 1 and 2 indicates number of odd days in an
Examples: ordinary year and a leap year respectively)
(i) Each of the years 1764, 1028, 1948, 1676, 2004 etc is a 124 odd days = 17 weeks + 5 days = 5 odd days.
leap year. Number of odd days in 100 years = 5.
(ii) Each of the years 400, 800, 1200, 1600, 2000, 2400 etc 2) Number of odd days in 200 years = (52) = 3.
is a leap year. 3) Number of odd days in 300 years = (53) = 1.
(iii) The years 2001, 2002, 2003, 2005, 1900, 2100 are not 4) Number of odd days in 400 years = (54+1) = 0.
leap years. Similarly, each one of 800, 1200, 1600, 2000 year etc has
Ordinary year: The year that is not a leap year is 0 (zero) odd days as they are multiples of 400.
called an ordinary year. An ordinary year has 365 days. Some Important points to remember:
In order to solve the questions on calenders, we use a 1) In every normal / ordinary year the first day (1st
concept called 'odd days'. January) and the last day (31 st December) are always
Odd day: The number of days more than a complete same. For example, if January 1st is Monday then
week are called odd days in a given period. December 31st is also Monday.
Lets discuss how to count the odd days in a given 2) In every leap year if the first day (January 1 st) is
period. Sunday, then last day (December 31st) will be it's next
Counting of odd days: day i.e. Monday.
To find the number of odd days in a given period, we 3) In every year, the calendar for the months April and
divide the total number of days with 7. The remainder July are always same.
obtained is the total number of odd days. 4) For every 400 years, the day repeats.
Examples: For example, if 14-April-1604 is Saturday, then 14-April-
1) How many odd days are there in 10 days. 2004 will also be Saturday.
Explanation: 7) 10 (1 5) The last day of a century cannot be either Tuesday or
7 Thursday or Saturday.
3 = Remainder 3 odd days.
2) How many odd days are there in 100 days.
Explanation: 7) 100 (14 Questions on day sequence/ calender are mainly 5
98 types.
2 odd days 1) Problems based on Total DayParticular Day.
3) How many odd days are there in an ordinary year? 2) Problems based on Leap Year.
Explanation: An ordinary year has 365 days. So, 3) Problems based on Particular DateDay.
7) 365 (52 4) Problems based on Same Calendar Year.
364 5) Problems based on Same DayDate of the Month.
1 odd days

Logon to www.CampusRecruitment.co.in and ask doubt with Question Id. 2.12


Find the details below to purchase the book.

How to order
Click Here

Click Here

Click Here

To purchase at us Click Here

www.CampusRecruitment.co.in
For Details, contact: sales@campusrecruitment.co.in
Or call us at: 07032206275
18)b; Odd day from 2024 to 2030 is 25)d; 1998 is not a leap year. So, write up to leap year
2024, 2025, 2026, 2027, 2028, 2029, 2030 before and after the given year including 1998.
i.e. 1996 1997 1998 1999 2000
2 1 1 1 2 1
Number of Odd days = 2 + 1 + 1 + 1 + 2 + 1 = 8 Now eliminate leap years. i.e. 1997, 1998, 1999.
8 Add the code (6) (11) (11).
Required Day = Given day + (remainder) 1997 1998 1999
7
6 11 11
= Tuesday + 1 = Wednesday
2009
The day on 01-Jan 2030 is Wednesday.
The sum corresponding to the given year is the answer.
Ask doubt with Question Id: 8000
The year 2009 will have the same calendar year 1998.
19)a; Here 600 is a century year. So, It should be
Ask doubt with Question Id: 8007
divisible by 400 to become a leap year. But it is not 26)a; Since, 2016 is a leap year. So, there are 2 odd days.
divisible by 400. So, 600 is not a leap year. Required day = Sunday + 2 = Tuesday.
Remaining all are leap years, because 2076 and 2084 Ask doubt with Question Id: 8008
are divisible by '4' and 2000 is divisible by 400. 27)b; No.of odd days from 26-Mar-2013 to 14-Oct-2013=
Ask doubt with Question Id: 8001 34
= 5 + 2 + 3 + 2 + 3 + 3 + 2 + 14 = (remainder)=6.
20)d; Given date 08May1986. 7
08+2 +85+ 21+0 116 So, 6 days after Tuesday is Monday.
Required day= = (remainder)= 4
7 7 Ask doubt using Question Id 8009
From day codes table 4 = Thursday. 28)c; If today is Monday, then day after tomorrow will
Ask doubt with Question Id: 8002 be Wednesday. We have to find the day 126 days ago of
21)e; Sahithya born 2 years, 4 months 2 days after Wednesday.
Alekhya born. i.e., Sahithya born exactly 2 years, 4 126
months and 3rd day from Alekhay's birthday. i.e., Required day = Wednesday (remainder)
7
Sahithya born on: 06 October 1994. = Wednesday 0
Required day=
06+1+93+23+0 123
= =4 (remainder) The required day is Wednesday it self.
7 7 Ask doubt with Question Id: 8010
From day codes table, 4 = Thursday.
Ask doubt with Question Id: 8003
22)a; In order to solve this, we should know the date of
first Sunday of January 2014. For this we have to find
the day of the 01-January-2014.
01+1+13+ 3+6 24
i.e. = (remainder)= 3=Wednesday
7 7
01-Jan-2014 is Wednesday.
So, first Sunday will be on 05-Jan. And Sundays fall on
5th, 12th, 19th and 26th of January 2014.
Ask doubt with Question Id: 8004
23)c;From the concepts, we know 100 years=5 odd days
Day after 100 years = Sunday + 5 = Friday.
Ask doubt with Question Id: 8005
24)a; Since, 2020 is a leap year, add 28 to get same
calendar year i.e. 2020 + 28 = 2048.
Ask doubt with Question Id: 8006

Logon to www.CampusRecruitment.co.in and ask doubt with Question Id. 2.19


DIRECTION SENSE TEST

Logon to www.CampusRecruitment.co.in and ask doubt with Question Id. 2.20


Find the details below to purchase the book.

How to order
Click Here

Click Here

Click Here

To purchase at us Click Here

www.CampusRecruitment.co.in
For Details, contact: sales@campusrecruitment.co.in
Or call us at: 07032206275
8)a; From option-a, (M 3 L) = M is the wife of L and 21)d; AC+B means A is the daughter of C who is the
(L 5 Z) = L is the father of Z. Then, M is the mother of Z. father of B. i.e. A is the sister of B.
Ask doubt with Question Id: 8059 Ask doubt with Question Id: 8072
9)c; From the family tree, U is the grandfather of S. 22)a; AC+B means A is wife of C who is the father of
P(+) = U(-) B. Then A is the mother of B.
Ask doubt with Question Id: 8073
R(-) Q(+) = V(-)
23)d; ACB means A is the brother of C who is the
T(-) S(+) daughter of B. It means, A is son of B.
Ask doubt with Question Id: 8060 Ask doubt with Question Id: 8074
10)a; From the above family tree, Q is the son of U 24)c; A+CB means A is the father of C who is the
Ask doubt with Question Id: 8061 daughter of B. It means B is the mother of C. Then A is
11)d; E and D are the cousins of F. husband of B.
A(-) B(+) C Ask doubt with Question Id: 8075
25)a; AC+B means A is the brother of C who is the
F(+) D(-) E(+) father of B. Then A is uncle of B
Ask doubt with Question Id: 8062 Ask doubt with Question Id: 8076
12)d; S is the brother-in-law of T.
Q P

S(+) R(-) = T(+)


Ask doubt with Question Id: 8063
13)c; AC+D+B means A is daughter of C who is the
father of D. D is father of B. i.e. A is sister of B's father
i.e. A is aunt of B.
Ask doubt with Question Id: 8064
14)d; P is the brother of Q and Q is the brother of R. So,
R may be the brother or sister of P.
Ask doubt with Question Id: 8065
15)c; Gita is aunt of the boy.
Uncle(+) Uncle(+)

Gita(-) Uncle(+)

Boy(+)
Ask doubt with Question Id: 8066
16)c; 3 children (Q, M, O).
P(+) N(+) = L(-)

Q(+) M(+) O(-)


Ask doubt with Question Id: 8067
17)d; There are four males.
Ask doubt with Question Id: 8068
18)d; L and O are females.
Ask doubt with Question Id: 8069
19)d; From given data, wife of P cannot be determined.
Ask doubt with Question Id: 8070
20)d; L is the mother of M.
Ask doubt with Question Id: 8071

Logon to www.CampusRecruitment.co.in and ask doubt with Question Id. 2.32


SYLLOGISMS
CONCEPTS Example: Statements: Some Note books are books
Questions on syllogism contains statements followed All books are papers
by conclusions. You have to analyze the given Conclusions: I. Some Notebooks are papers
statements carefully and find which of the conclusions II. No papers is notebook
logically follow. Explanation: The possible Venn diagram for the given
Each statement of syllogism contains of three parts. statements is as follows:
They are, subject, predicate and copula.
Subject is that about which something is said. N B
Pedicate is that part of the statement that which affirms P
is denied about the subject. From the given statements, statement-I follows from
Copula is the word of the statement which denotes the the above diagram but statement-II does not follow.
relation between the subject and predicate. Ask doubt with Question Id: 8078
Example: Consider the statement, 'woman is talented'. 2) Rules and Application Method:
Here an information about woman is given. So, woman The following rules are very useful while solving
is the subject. 'talented' is the quality affirmed for this problems using this metod.
subject. So it is the predicate. 'is' denotes the relation all + all all
between subject and predicate. So, it is copula. all + no no
Here we will discuss three types of methods to solve some + all some
the questions on syllogism. all + some no conclusion
1) Diagramatic method no + no no conclusion
2) Rules and Application method some + some no conclusion
3) Numbering and Apply method some + no some-not
1) Diagramatic Method: no + all reverse of some-not
To solve the syllogism questions in this method, you no + some reverse of some-not
have to represent the given statements in the form of a some-not/reverse of some-not + anything = no conclusion.
diagram. Implication Statements:
In syllogisms, frequently we come across the terms like
1) All Some
all, some and no, not etc.
If 'all As are Bs' then 'some As are Bs' is also true.
Example: 1) All papers are pens.
2) Some Some
If the above statement are If 'some cats are rats' then 'some rats are cats' is also true.
represented in a diagram, papers 3) No No
will be in inner circle and pens will Paper If 'no car is bus' then 'no bus is car' is also true.
be in outer circle. i.e. For either-or option: If one statement is positive (i.e.
Pens
starts with all/ some) and the other statement is negative
(i.e. starts with no) and if they both have same objects
2) Some papers are pens.
then the answer will be those two conclusions with
either-or words.
Paper Pen Example: 1) Statements: Some keys are locks
All locks are doors
Some
Conclusions: I. All keys are doors
3) No paper is pen. II. Some keys are doors
'no' indicates there is no relations III. Some keys are not doors
exists between subject and Paper Pen a) only conclusion-(I) follows
predicate of the given statement. b) only conclusion-(II) follows
Hence, the two circles will not meet No c) only conclusion-(III) follows
each other. d) both conclusion-(I) and (III) follows
e) None of the given conclusions follow

Logon to www.CampusRecruitment.co.in and ask doubt with Question Id. 2.33


Find the details below to purchase the book.

How to order
Click Here

Click Here

Click Here

To purchase at us Click Here

www.CampusRecruitment.co.in
For Details, contact: sales@campusrecruitment.co.in
Or call us at: 07032206275
CHARACTER PUZZLE
CONCEPTS Explanation: From (a), 8 3 + 6 = 30
In this type of questions, a figure or a matrix is given in From (b), 9 4 + 8 = 44
which some numbers are filled according to a rule. A So, from (c), 12 6 +7 = 79
place is left blank. You have to analyse the given Ask the doubt using Question Id: 8209
character or number and find out the missing number 5) What number will replace the question mark ?
or letter from the given possible answers which may be
3 6 9 7 8 6
filled in the blank space.
CONCEPTUAL EXAMPLES 110 159 ?
1) Which number will replace the question mark? 4 7 5 2 3 5
(a) (b) (c)
5 3 6 8 9 4 a) 229 b) 129 c) 329 d) 439 e) 339
9 20 2 19 7 ? Explanation: From (a), 32 + 62 + 42 + 72 = 110
From (b), 92 + 72 + 52 + 22 = 159
(a) (b) (c)
So, from (c), 82 + 62 + 32 + 52 = 129
a) 236 b) 336 c) 286 d) 386 e) 436
Ask doubt with Question Id: 8210
Explanation: [378] + [236] = 204
6) Which number will replace the question mark?
[465] + [872] = 232
12 19 16
Similarly, [458] + [673] = 286
4 3 ?
Ask the doubt using Question Id: 8206
6 3 8
2) Which number will replace the question mark ?
8 19 4
7
3 8 4 6 5 4 5 8 a) 8 b) 16 c) 4 d) 2 e) None of these
Explanation: From Column-I: (12 4) 6 = 8
204 232 ?
From column-II: (19 3) 3 =19
2 So, from column-III: (16 ?) 8 = 4 16? = 32 ? = 2.
3 6 8 7 2 6 7 3
a) 23 b) 18 c) 22 d) 21 e) None of these Ask the doubt using Question Id: 8211
Explanation: 7) Which number will replace the question mark?
From figure (a), 9 + 5 + 3 = 17 17 + 3 = 20 3 9 6
From figure (b), 2 + 6 + 8 = 16 16 + 3 = 9 55 32 ?
8 7 12
From figure (c), 7 + 9 + 4 = 20 20 + 3 = 23
(a) (b) (c)
Ask the doubt using Question Id: 8207
a) 108 b) 46 c) 64 d) 104 e) can't be determined
3) which number will replace the question mark?
Explanation: From (a), 82 32 = 55
4 6 3 5 8 2 From (b), 92 72 = 32
So, from (c), 122 62 = 108
Ask the doubt using Question Id: 8212
8) Which character will replace the question mark?
100 52 ?
(a) (c) A5 F10 K15
(b)
a) 81 b) 84 c) 516 d) 514 e) 86 B16 G21 L26
Explanation: From (a), 43 + 62 = 64 + 36 = 100
C27 H32 ?
From (b), 33 + 52 = 27 + 25 = 52
So, From (c), 83 + 22 = 512 + 4 = 516 a) M37 b) N36 c) O37 d) M36 e) M38
Ask the doubt using Question Id: 8208 Explanation:
4) Which number will replace the question mark? From column-I, ABC (5+11+11)
8 9 12 From column-II, FGH (10+11+11)
30 From column-III, KLM (15+11+11)
44 ?
3 6 4 8 6 7 The character M37 will replace the question mark
Ask the doubt using Question Id: 8213
a) 79 b) 89 c) 99 d) 69 e) 109
Logon to www.CampusRecruitment.co.in and ask doubt with Question Id. 2.54
Find the details below to purchase the book.

How to order
Click Here

Click Here

Click Here

To purchase at us Click Here

www.CampusRecruitment.co.in
For Details, contact: sales@campusrecruitment.co.in
Or call us at: 07032206275
STATEMENTS AND ARGUMENTS
CONCEPTS argument-(I) does not hold strong. Hence, only
In this type of questions, the statement deals with all argument-(II) is strong.
general aspects of day to day life which may include Ask doubt with Question Id: 8322
socio economic, scientific, political issues etc. A 2) Statement: Should taxes on air conditioners be
statement is followed by two arguments. One supports further increased?
the statement by pointing out the positive aspects and Arguments:
the other deny the statement by pointing out it's I. Yes, air conditioner is a luxury item and rich people
negative impact. You have to analyze given statement, can only buy them.
arguments and decide which of the arguments strongly II. No, air conditioners are bought by financially
supports the statement by giving an appropriate backward sector also.
opinion on the subject. Read the given arguments in Explanation: Generally, taxes on any commodities or
the question and discard them if they are ambiguous, goods doesn't depend on the financial position of the
disproportionate, irrelevant, comparative, simplistic. individuals so, both the arguments does not hold
1) Ambiguous: If the given arguments does not have a strong. Hence, option-d is correct choice.
clear reason or if it is not contextual or not expressing Ask doubt with Question Id: 8323
its opinion whether supporting or not. Such an 3) Statement: Should Indian software professionals
argument should be discarded. who are working abroad be called back?
2) Disproportionate: If the given arguments are too Arguments:
large or too small in comparison with given statements. I. Yes, they must serve the mother land first and forget
This kind of arguments can be discarded. about high pay scales or facilities etc.
3) Irrelevant: If the given arguments are irrelevant to II. No, we have adequate talent here, let them stay
the context of the given statements, they can be according to their will and wish.
discarded. Explanation: The demands of an individual are as
4) Comparative: If the argument do not state the important as the demands of motherland. So,
reasons for why the proposed action is implemented argument-(I) is not strong. Argument-(II) is weak
and its consequences. Such arguments can be because of its complacent attitude. Hence, option-d is
discarded. correct choice.
5) Simplistic: If the given arguments so not have Ask doubt with Question Id: 8324
sufficient information to support the given statements, 4) Statement: Should education to women be made
they can not be considered. free in India?
CONCEPTUAL EXAMPLES Argument:
Direction: Each of the following examples consists of a I. No, this will weaken our present social structure.
statement followed by two arguments (I) and (II). Give II. Yes, this is the only way to bring back glory to
your answer as, Indian woman hood.
a: if only argument-(I) is strong. Explanation: Argument-(I) is strong. It is links,
b: if only argument-(II) is strong. providing free education to women with weakening of
c: if either argument-(I) or (II) is strong. social structure, which is not sensible. Argument-(II)
d: if neither of the arguments is strong. also weak because of the term 'only'. Hence, option-d is
e: if both the arguments are strong. correct choice.
1) Statement: Should number of holidays be increased Ask doubt with Question Id: 8325
to private employees? 5) Statement: Are joint families better than small
Arguments: families.
I. Yes, because employee satisfaction will be better. Arguments:
II. No, it will lead to decreased productivity of private I) Yes, joint families provide more security and unity
organizations. and also reduce the burden of work.
Explanation: Though employee satisfaction is II) No, small families ensure more freedom.
important but this will adversely affect the
productivity and revenue of the organization. So the

Logon to www.CampusRecruitment.co.in and ask doubt with Question Id. 2.68


Find the details below to purchase the book.

How to order
Click Here

Click Here

Click Here

To purchase at us Click Here

www.CampusRecruitment.co.in
For Details, contact: sales@campusrecruitment.co.in
Or call us at: 07032206275
VERBAL ABILITY
PARTS OF SPEECH
Parts of speech in a language mean the role or part played by a word in a sentence. This is similar to the roles we
play in our lives. In one place you might be a student, in another a friend, yet in another, if you are working, you
might be a boss or a subordinate. We also play different roles at home when we interact with different people. In
all the roles we play, our role and interaction depends upon the relationship with the people with whom we are
interacting. Similarly, words are categorized according to the role or part they play in a sentence. Seeing a word,
we cannot categorize it as a noun/ pronoun/ verb/ adjective etc. In order to categorize a word, we need to know
what role it plays in the sentence. Words are normally categorizes into 8 parts of speech. Here we give you the
categories with some examples:

Part of Speech Function Example Words Example Sentences


Name of a person, place, thing Girl, Manchester,
NOUN Gandhiji loved peace.
or quality Gandhiji, peace, honesty
PRONOUN Words used instead of a noun He, she, our, theirs, my ... She is our teacher.
Beautiful, big, tall, The girl is tall.
ADJECTIVE Words used to describe nouns.
awesome Gandhi was a great leader.
Be verbs, have verbs and
do verbs, She is my friend.
Words which tell the state of a am, Is, are, was, were, He paints well.
VERB
thing, possession and action. have, has, had She works meticulously.
Do, does, did
work, talk, walk
Words which tell us where,
when and how an action takes fast, very, sincerely, She works fast.
ADVERB
place, as well as to what degree properly, quite .. This train is quite fast.
an action takes place..
Theres a park near my
Words which tell us the position In, on, between, under,
house.
PREPOSITION or relationship between two for, near, by, with
Your pen has fallen under
nouns in a sentence.
the chair.
And, but, or, neithernor, Bread and butter is taken
Words which join other words
eitheror, so when, by many for breakfast.
CONJUNCTION or sentences to make language
while, who whom. He started early but could
more concise
not reach on time.
Wow! Great! Spectacular! Wow! What a great shot!
Words used to express sudden
INTERJECTION Awesome! What a spectacular
feelings and expressions.
performance!

Lets see these parts of speech in detail:

Logon to www.CampusRecruitment.co.in and ask doubt with Question Id. 3.2


NOUN
Nouns are commonly called naming words. i.e. names e.g.: advice, news, furniture.
of persons, places, things, quality and material. The news is good.
Nouns are of five kinds: Wooden furniture is expensive.
Proper nouns are names of persons, places, rivers Your advice has been taken.
and mountains. 7. Units of counting hundred, thousand, pair, dozen,
e.g.: Ravi, India, Hyderabad, Godavari, Himalayas. score etc, retain their singular form when used after
These always start with a capital letter. numbers.
Common nouns are names given commonly for e.g.: She bought two score papers for her project.
places, persons, animals, things etc. (not two scores)
e.g.: village, town, boy, girl, dog, pen, pencil, trees, etc I need three dozen eggs.
Collective noun is the name given to a group of Four hundred chairs were ordered for this hall.
animals, things and people. 8. Some nouns are used only in the plural:
e.g.: a staff of teachers, a school of fish, a flock of birds, a) Names of certain instruments which are used as a
a bunch of grapes etc. pair: scissors, pincers, pliers, tongs, spectacles.
Abstract noun is the name given to things which b) Names of certain articles of dress: trousers, pants,
cannot be seen or felt, but thought of. shorts.
e.g.: happiness, wisdom, intelligence, patience etc. c) Names of certain diseases: mumps, measles.
Material nouns refer to the names of material of d) Names of certain games: billiards, draughts.
which products are made. e.g.: His new spectacles are very expensive.
e.g.: wood, leather, silk etc. These tongs are damaged.
Apart from this, nouns are also classified according to His trousers are short.
number and gender. Billiards is an interesting game.
Number: Gender: There are four types of genders.
All nouns which can be counted are called countable. Masculine gender refers to males.
e.g.: books, girls, pens, rooms, villages etc. e.g.: man, boy, lion, conductor.
Nouns which cannot be countable are called Feminine gender refers to females.
uncountable. e.g.: woman, girl, lioness, conductress.
e.g.: milk, oil, hair, stars, news, advice, information etc. Common gender refers to both males and females.
1. Countable nouns can be either singular or plural. e.g.: child, teacher, engineer, servant.
Singular denotes one and plural denotes more than Neuter gender refers to lifeless or inanimate objects.
one. Plurals are formed by adding- s, es or ies. e.g.: table, book, chair etc.
e.g.: Girl Girls, Church Churches, Lily Lilies. Troublesome Rules and Confusing Areas
2. Words ending in f and fe change into ves. 1. Failure in identifying nouns leads to common
e.g.: Life Lives, Wife Wives mistakes. Nouns generally end in:
3. Plurals are also formed by changing the middle -ness - kindness, sadness, happiness
vowel. -tion - nation, ration, action
e.g.: Man Men, Foot Feet, Mouse Mice -ssion - commission,omission, permission, mission
4. Words ending in y preceded by a consonant change -er - player, driver, scavenger
into ies to form plurals.
-ice - justice, practice
e.g.: Story Stories, Baby Babies, Lady Ladies
-sion - division
5. Words of foreign origin form their plurals in a
-ance - finance
different way.
-ment - government, increment
e.g.: Alumnus Alumni, Curriculum Curricula,
-hood - neighborhood, childhood
Medium Media.
There are certain exceptions to these rules which have -dom - freedom, kingdom, martyrdom
to be memorized. -cy - hesitancy, piracy, conspiracy
e.g.: Ox Oxen, Roof Roofs, Safe Safes -ist - florist, chemist, dentist
6. Some nouns do not have a plural form and are -ity - charity, enmity, locality
always used with a singular verb. -ship - friendship, workmanship
Logon to www.CampusRecruitment.co.in and ask doubt with Question Id. 3.3
2. Plurals of possessive nouns, which end in the letter 14. He ate two breads for breakfast. (two slices of bread.
s, are formed by adding an apostrophe after the word. bread in not countable)
e.g.: Srinivas desk, students notebooks, girls hostel. 15. Ten miles are a long distance. (is)-a unit of distance
3. In case of joint possession, only the last word shows so used as a singular.
possession.
e.g.: Rani and Rajus dog.
4. In compound nouns only the last word shows
possession.
e.g.: Father-in-laws house, Editor-in-chiefs office.
5. Material nouns are not used in plural numbers.
e.g.: This furniture is made of wood. (not woods)
6. A collective noun usually takes a singular verb and is
substituted by a singular pronoun.
e.g.: The team has performed well. It has scored the
highest number of runs in this series.
But, if the members of the group act as individuals, not
as a group, then the collective noun takes a plural verb
and is substituted by a plural pronoun.
e.g.: The council has submitted its report.
(Here, the council is used as singular. So, singular verb,
has is used)
The council are divided on the outcome of the issue.
(Here, the council is used as plural. So, plural verb, are
is used).
Correction of Sentences
1. The crowd were very big. (was)
2. The chair is made of woods. (no plural for material
nouns)
3. My spectacles is broken. (spectacles is always plural
so verb should be are. Spectacle also means scene).
4. The Indian Army are helping the people of Nepal
affected by earthquake. (is)
5. Sachin Tendulkar scored more than twenty
thousands runs in test cricket. (thousand - units of
counting retain singular form).
6. His trouser is new. (trousers - always plural)
His trousers are new Correct.
7. He has a lot of properties. (property - abstract nouns
not used in the plural)
8. He has bought new furnitures. (furniture-no plural)
9. She has got her hairs cut. (hair - not countable)
10. You must brush your teeths regularly.
(teeth itself is the plural form)
11. The childrens of this school have performed well in
the public examination. (Children - itself is the plural
form)
12. His wives purse was stolen. (wifes)
13. I am visiting the New Delhi. (no article needed for
proper names)

Logon to www.CampusRecruitment.co.in and ask doubt with Question Id. 3.4


Find the details below to purchase the book.

How to order
Click Here

Click Here

Click Here

To purchase at us Click Here

www.CampusRecruitment.co.in
For Details, contact: sales@campusrecruitment.co.in
Or call us at: 07032206275
SUBJECT - VERB AGREEMENT
1. All subjects must agree in number and person with The council has chosen its president.
the verb. The council are divided on the issue of making Aadhar
e.g.: The students of the primary section are going on a card mandatory.
picnic. 11. Some nouns which are plural in form, but singular
The quality of these mangoes is very good. in meaning, take a singular verb.
2. Two or more singular subjects connected by and e.g.: The news is true.
usually take a verb in the plural. Civics is important for people who wish to enter the
e.g.: Sheela and Ragini are here. civil services.
Wealth and generosity dont go together. 12. When the plural noun is a proper name for some
3. If two singular nouns refer to the same person or single object or some collective unit, the verb should be
thing the verb must be singular. singular.
e.g.: My best friend and confidant has come. e.g.: The United States has a big army.
(a confidant is a friend in whom you can confide your Gullivers travels was written by Swift.
secrets) 13. When a plural noun denotes some specific quantity
The Chairman and Managing Director is going to or amount considered as a whole, the verb is generally
address the employees. singular.
4. If two subjects together express one idea, the verb is e.g.: Ten miles is not a short distance.
singular: Fifty thousand rupees is a large sum.
e.g.: Slow and steady wins the race. 14. A common blunder is to leave the Participle without
Bread and butter is his only food. proper subject.
5. Words joined to a singular subject by with, together e.g.: Sitting on a gate, a scorpion stung him. (wrong)
with, in addition to, or as well as are parenthetical and so (who was sitting on the gate)
the verb should be in singular. While he was sitting on the gate a scorpion stung him.
e.g.: The house with its contents was insured. (correct)
The price of silver as well as gold has fallen. Being a hot day, I stayed at home. (wrong)
6. Two nouns qualified by each or every even though (who or what is the hot day)
connected by and require a singular verb. It being a hot day, I stayed at home. (correct)
e.g.: Each senior member was honored. Correction of Sentences
Every man and woman was present at the festival. 1. She is going to temple everyday.
7. Two or more singular subjects connected by or, (simple present; so goes)
neither.. nor, either..or take a verb in the singular. 2. These school children needs books and pencils. (need)
e.g.: Neither he nor I was there. 3. When he is playing cricket, he fell down.(was playing)
Either he or I am mistaken. 4. She has completed her graduation last year.
8. When the subjects joined by or, nor are of different (remove has)
numbers, the verb must be plural and the plural subject 5. They are living here since 2010. (have been living)
must be placed before the verb. 6. When the ambulance came the patient died.(had died)
e.g.: Rana or his brothers have done this. 7. They are having the same car for 10 years. (have had)
Neither the Principal nor the teachers were present. 8. She will leave for Mumbai tomorrow. (is leaving)
9. When the subjects joined by or, nor are of different 9. We have gone to Chennai last week. (have been to)
persons, the verb agrees in person with the one nearest 10. She is hearing to carnatic music. (listening)
to it. 11. She never visited Charminar before.
e.g.: Either he or I am mistaken. (had never visited)
Neither you nor he is to blame. 12. Your parcel didnt come yet. (hasnt)
10. A singular verb should be used with a collective 13) He sleeping while his wife cleaning the house.
noun, when the collection is thought of as one whole. (was sleeping was cleaning)
A plural verb should be used with a collective noun 14. I am liking mangoes. (like)
when it refers to the individuals who comprise it. 15. He is always forgetting my phone number. (forgets)
e.g.: The crew was large.
The crew were taken prisoners.
Logon to www.CampusRecruitment.co.in and ask doubt with Question Id. 3.13
Find the details below to purchase the book.

How to order
Click Here

Click Here

Click Here

To purchase at us Click Here

www.CampusRecruitment.co.in
For Details, contact: sales@campusrecruitment.co.in
Or call us at: 07032206275
CONJUNCTION
A conjunction is a word which merely joins together be booked.
words or sentences, they do no other work. If you had asked me earlier, I could have helped you.
e.g.: She and her friends are visiting us. 6. Comparison:
He came early but couldnt complete the work. e.g.: She is as tall as her sister.
Some conjunctions are single and some conjunctions He is richer than I am.
are used in pairs. Some of these are: 7. Concession:
either or, neither nor, not only but also, though yet, e.g.: Although he worked hard, he could not get a state
whether or etc. these conjunctions which are used in rank.
pairs are called Correlative Conjunctions or just Though he is strong, he is unable to do this work.
Correlatives. Troublesome Rules and Confusing Areas
Some compound expressions are also used as 1. The most common mistake is the placement of the
conjunctions and these are called compound conjunction. The conjunction should be placed just
conjunctions. Some of these are: before the clause it introduces.
even if, as though, as well as, as if, as soon as, so that, e.g.: It is raining because he has not come. (incorrect)
in order that etc. He has not come because it is raining. (correct)
Conjunctions are divided into two classes: 2. 'Scarcely' is followed by 'when'.
Coordinating and Subordinating. e.g.: Scarcely had we entered the house when it started
Coordinating Conjunctions bring together two raining.
independent statements or two statements of equal 3. 'No sooner' is followed by 'than'.
rank or importance. The main coordinating e.g.: No sooner had she got her results than she got a
conjunctions are: job
and, but, or, nor, also, either-or, neither-nor. 4. 'Neither' is followed by 'nor'.
e.g.: He is slow but steady. e.g.: He is neither intelligent nor hardworking.
The thieves broke the door and entered the house. 5. While using 'not only . . . but also', the verb must
You must return the book tomorrow or pay the fine. agree with the noun or pronoun mentioned second.
Subordinating conjunctions bring together two e.g.: Not only the students but the teacher were also
statements or clauses, one of which is dependent on the injured. (incorrect)
other. The chief subordinating conjunctions are: Not only the students but the teacher was also injured.
after, because, if, that, though, although, till, before, unless, (correct)
as, when, where, while. Correction of Errors
e.g.: You will pass if you work hard. 1. He is sincere and also hardworking.
He didnt speak up because he was afraid. (not only - but also)
Though he was ill, he attended the meeting. 2. She asked me whether I had a pen or not.
He came after I had left. (or not can be omitted)
Subordinating conjunctions may be classified 3. He did not come or sent a message. (neither nor)
according to meaning or function: 4. He not only broke the glass, but threw it away.
1. Time: (not only. but also)
e.g.: I knew him before he came here. 5. Both he and I contributed to the fund. (no error)
I waited till the train arrived. 6. No sooner had the bell rung then the students ran
2. Cause or reason: out. (replace then with than)
e.g.: Since you say so I must believe it. 7. She is taller as her sister. (as tall as)
He did not come because you did not call him. 8. He worked hard and could not get a state rank. (you
3. Purpose: can use but instead of 'and', or start the sentence with
e.g.: We eat that we may live. Although)
He deserved the prize for he had worked hard. 9. There is a bus strike because she is not coming.
4. Result or consequence: (she is not coming because there is a bus strike)
e.g.: He was rude so he was punished. 10. He will return the money on the 1st or 2nd.
5. Condition: (He will return the money either on 1st or 2nd)
e.g.: Unless you bring your Passport, the tickets cannot
Logon to www.CampusRecruitment.co.in and ask doubt with Question Id. 3.22
Find the details below to purchase the book.

How to order
Click Here

Click Here

Click Here

To purchase at us Click Here

www.CampusRecruitment.co.in
For Details, contact: sales@campusrecruitment.co.in
Or call us at: 07032206275
SENTENCE CORRECTION or SPOTTING ERRORS
CONCEPTS To score well in the above sections, you need to
Sentence correction questions are designed to test your know Standard English Grammar. You must be able
ability to identify written English that is grammatically to recognize the various Parts of Speech and identify
correct. They also test your ability to understand the the way they are used incorrectly in test question.
essential message conveyed in that sentence. Therefore, Mainly, your attention should be focused on tenses of
understanding the essential and discarding the verbs, word order, word form, and agreement of the
unimportant or non-essential is the key point to be verb with the subject, difference between principal verb
focused while attending to these type of questions. and Auxiliary verb, proper usage of preposition. You
Questions on spotting errors/ sentence correction are must also have a solid understanding of the different
usually based different grammatical rules. We have idiomatic phrases and the link between one clause and
different types of grammatical errors. You have to the other. i.e. principal clause and sub-ordinate clause.
concentrate chiefly on the following kinds of errors.
1) Errors of Subject Verb Agreement. Strategies to solve questions on choosing
2) Errors based on the wrong usage words. grammatically correct sentences.
3) Errors in the use of Nouns and Pronouns, Adjectives, a) The first thing to do is to go through all the four
Adverbs, Conjunctions, Prepositions etc. sentences carefully. Sometimes there may be multiple
4) Errors in the use of Tenses. errors in a sentence. Therefore while choosing the
5) Errors on active and passive voice. correct sentence; you have to be careful.
6) Errors in the usage of Articles b) While reading the options you may find one or two
7) Errors in the usage of Reported Speech. sentences with glaring grammatical mistakes.
All the above topics are discussed in the previous Obviously, what you should do is to short list your
chapters. options. Then closely concentrate on the one or two
Types of sentence correction questions: short listed options out of the four given.
Spotting the Errors: c) Do not look for spelling errors or errors is the use of
In this type of question, a sentence is given with certain capital letters and punctuation marks.
parts have been underlined or marked a, b, c and d. EXERCISE-I
one of these underlined or marked parts may contain Each of the questions given below consists of four
and error or may not be acceptable in standard written sentences on a topic numbered a to d. There may be
communication. You have to find which part has an an error in any one part of the sentence. Read the
error. sentence and Identify the part which has the error
Choosing the grammatically correct sentences: and write it as the answer. If there is no error, mark
In this type of question, four sentences are given and the answer as (e).
we are asked to choose the grammatically correct 1. (a) If he worked hard /(b) he would have /(c) got
sentence. There is no underlined part so you have to through the exam /(d) which he wrote recently.
observe the entire sentence for its accuracy and 2. (a) She told me /(b) that she will meet me /(c) the
grammar. next day /(d) if she had time.
Choosing the best alternative: 3. (a) He usually /(b) goes to college /(c) by walk /(d)
This is a different type of question where a part of the because he stays close by.
sentence is high-lighted or underlined. You have to 4. (a) After he returned back /(b) from U.S /(c) he has
choose the best alternative from among the four given started /(d) his own factory.
sentences. 5.(a) The students /(b) work /(c) meticulously
Inappropriate usage: throughout the year /(d) doesnt they.
Here, the different usages of a word/ phrases are 6. (a) She could not give the exam /(b) since she lacked
tested. You have to choose the option in which the /(c) the requisite attendance /(d) required by the
usage is inappropriate or incorrect. University.
7. (a) He hoped /(b) to top the exam /(c) but could not
be able to do so /(d) because of ill-health.

Logon to www.CampusRecruitment.co.in and ask doubt with Question Id. 3.24


Find the details below to purchase the book.

How to order
Click Here

Click Here

Click Here

To purchase at us Click Here

www.CampusRecruitment.co.in
For Details, contact: sales@campusrecruitment.co.in
Or call us at: 07032206275
13)c; Convention means tradition or a practice. It is 23)c; We do not make 'a large mistake'. We make 'a big
appropriately used in sentences-a, b, d. Sentence-c mistake'. Hence, sentence-c is incorrect.
should be 'It was a well established convention'. The word Ask doubt with Question Id: 2089
convention does not collocate with word lay. 24)d; Fit refers to size and shape whereas suit refers to
Ask doubt with Question Id: 2079 style, colour etc. Hence, sentence-d should be 'Red and
14)d; Pose means posture or to put forward an idea. We yellow are colours that fit colour that suits her very well'.
do not say 'poses a heavy burden'. We say 'places/ puts a Ask doubt with Question Id: 2090
heavy burden'. Pose does not collocate with burden. 25)c; The word Indulge does not collocate with the
Hence, sentence-d is correct choice. word violence. Sentence-c should be 'to resort to
Ask doubt with Question Id: 2080 violence'.
15)d; To talk about getting knowledge we use the word Ask doubt with Question Id: 2091
tell. Therefore, sentence-d should be 'you can tell from 26)d; In sentence-d, the correct usage is 'to open a
his accent'. discussion' but not 'to raise a discussion'.
Ask doubt with Question Id: 2081 Ask doubt with Question Id: 2092
16)d; Control generally means to command or to
restrict, govern etc. The sentence-d should be 'I took
my car to the garage and asked then to have a look at /or
check the steering'.
Ask doubt with Question Id: 2082
17)a; In sentence-a, 'you don't need to pay for emergency
calls in England' is correct. We cannot use 'need' as a
modal form of verb to talk about habitual, general
necessity.
Ask doubt with Question Id: 2083
18)b; Sentence-b is incorrect. It should be 'The road out
of our village goes up a steep hill'. A street is a road with
houses on either side. We use street for roads in towns
or villages but not for the roads which are out of the
towns or villages.
Ask doubt with Question Id: 2084
19)b; Watch is typically used to talk about experiences
that are going on or going to happen where as 'see' is
used to talk about the whole of a performance, play,
cinema etc. Hence, sentence-b should be 'Have you ever
seen series, The great Predator'.
Ask doubt with Question Id: 2085
20)a; Descend means to fall or move down. Sentence-a
should be 'calm descended on the crowd'. If a group of
people descend on a place, they arrive, usually without
warning or without being invited.
Ask doubt with Question Id: 2086
21)b; Sentence-b should be 'let's forget this problem'.
Ask doubt with Question Id: 2087
22)a; The correct usage is 'thanks for bringing me here'.
We use the word bring for movements to the place
where the speaking is but we use the word take for
movements to other places. Hence, sentence-a is
wrong.
Ask doubt with Question Id: 2088

Logon to www.CampusRecruitment.co.in and ask doubt with Question Id. 3.33


Find the details below to purchase the book.

How to order
Click Here

Click Here

Click Here

To purchase at us Click Here

www.CampusRecruitment.co.in
For Details, contact: sales@campusrecruitment.co.in
Or call us at: 07032206275
READING COMPREHENSION
Reading is a skill which has other sub-skills included in One of the most effective ways of reading in order to be
it. It is not mere recognition of the words, it also able to comprehend quickly is the SQ3R method:
includes being able to understand, comprehend and Scan
Question
respond, if questioned about the text. Many languages
Read
share the same script: Hindi, Punjabi, Telugu, Kannada.
Review
For that matter all European Languages like English, Recite
French, Spanish and German have the same script. Scanning provides a rapid overview. Many well
Being able to read a script does not ensure written books follow logical outlines that can orient the
understanding the script. Another important aspect is reader to the subject matter.
familiarity with the content. If you are not in the habit Questioning is a natural, instinctive, second step that
most winners follow. In the scanning process, certain
of reading, if you are not aware of whats happening
questions naturally arise. These should be noted in a
around you, then even the simplest of scripts will seem
short list of questions to be answered through reading.
like Greek and Latin. The most important point is The questioning procedure helps the reader stay
READ, READ and READ. focused.
1. Spend a few minutes a day reading at a faster than First, determine the main idea from the title, the first
comfortable rate (about 2 to 3 times faster than your paragraph, and the last paragraph.
normal speed). Use your hand or an index card to Second, determine if a large subject is divided into
smaller subjects with some outlining scheme.
guide your eyes down the page. Then time yourself
Underline key words or take notes to the side what
reading a few pages at your normal speed. the purpose of the paragraph is. i.e. cause, effect,
2. If you have poor concentration when reading, reason, example, definition, instructions, background
practice reading for only 5 to 10 minutes at a time and info, etc. Dont worry if you cant do that for all and
gradually increase this time dont spend too much time trying to identify each
3. As we read, our eyes move along the line in a series paragraph.
of jerky movements, stopping at each word. Fast Read for Author's Main Idea and Primary Purpose.
readers usually take in 3-4 words in each movement At the end of reading, ask yourself questions like:
that their eye makes. The more words you can take in
What was the passage about? What was author's
with each movement of the eye, the faster your reading
motive in writing all this?
will be. Try to avoid focusing on every word, but rather
look at groups of 2 to 3 words. Don't over read. Skip examples, dates, lengthy
e.g.: The above sentence could be read as: names, any details which can be referred in case
Try to avoid/ focusing on every word/ but/ rather look something is asked explicitly.
at/ groups of 2 to 3 words. Don't go for choices which hold true only for one
4. Read more! 15 min a day of reading an average size
part of the author's argument.
novel equals 18 books a year at an avg reading speed!
Finally, review as often as necessary to keep focused.
5. Spend a few minutes a day reading at a faster than
Outlining and note-taking often help.
comfortable rate (about 2 to 3 times faster than your
Once you start to become an effective reader, you will
normal speed). Use your hand or an index card to
find that you are also becoming a faster reader.
guide your eyes down the page. Then time yourself
With these tips your reading skills are sure to improve
reading a few pages at your normal speed. You'll find
that often your normal reading speed will increase.
Being a voracious reader is just not enough. In
examinations where reading skills are tested, the ability
to read and comprehend fast is needed. Here are some
tips to tackle the reading section. In order to improve
your reading speed, follow these steps.

Logon to www.CampusRecruitment.co.in and ask doubt with Question Id. 3.61


Find the details below to purchase the book.

How to order
Click Here

Click Here

Click Here

To purchase at us Click Here

www.CampusRecruitment.co.in
For Details, contact: sales@campusrecruitment.co.in
Or call us at: 07032206275
49)b; From the concluding statement, we can say 60)c; According to the passage, a single cycle of a
Durkheim was vindicated on all counts. motion completed in the shortest time is the Earth's
Ask doubt with Question Id: 2373 rotation on its axis. Because remaining all other
50)c; From paragraph-2, it is clear that all nutrient motions take more time to complete its task.
materials and waste products exchanged between the Ask doubt with Question Id: 2384
organs and the blood must traverse peri-vascular 61) In the first sentence of the passage, the author is
spaces occupied by connective tissue. most concerned with attacking a particular approach to
Ask doubt with Question Id: 2374 the social sciences.
51)c; The first line of the passage itself encircles the Ask doubt with Question Id: 2385
originality of the connective tissues to the Embryo. 62)a; According to scientism there is only one truth, the
Ask doubt with Question Id: 2375 truth of science and the methods of physical science
52)c; First line of paragraph-3 states, mesenteries are can thus be applied to other fields of enquiry, like the
thin sheets from which organs are suspended. social sciences. Hence, option-a is correct choice.
Ask doubt with Question Id: 2376 Ask doubt with Question Id: 2386
53) Option-c is the correct answer. 63)c; Option-a is not a proper explanation for the
Ask doubt with Question Id: 2377 question. Option-b is also not related to the question as
54)d; From paragraph-3, adipose tissue a connective well as to the passage. Option-d directly denies the
tissue in which fat is stored. Hence, option-d is correct. fact. But option-c is suitable to the passage where the
Ask doubt with Question Id: 2378 author states about it.
55)b; The tissue which enables smooth gliding Ask doubt with Question Id: 2387
movements of neighboring surface is cartilage. Option- 64)b; According to the author, causes and effects in the
b is correct. social world are difficult to identify or predict. He has
Ask doubt with Question Id: 2379 given the reason about this in the last paragraph of the
56)d; Option-a says the study of the structure and passage ---'Any single cause in the social.....'.
diseases of the brain and all the nerves in the body is Ask doubt with Question Id: 2388
not related to the passage. Likewise option-b says the 65)c; This passage is a lucid and coherent criticism
substances that you take into your body as food and regarding scientism. The author lucidly explains the
the way that they influence your health is also not the technical theorems about the term scientism. Obviously
thematic concern of the passage. Similarly option-c the author is not going to scrutinize, approve, or even
says simple physical exercises that are done to make dismiss the matter.
the body firm, able to stretch easily and more Ask doubt with Question Id: 2389
attractive. But option-d is rightly to be the answer as it
says (the scientific study of) the way in which the
bodies of animals and plants work. Hence, option-d is
correct.
Ask doubt with Question Id: 2380
57)a; The passage is all about the Earth's motions. In
the beginning lines of the passage itself it is clearly
evident.
Ask doubt with Question Id: 2381
58) With the basic understanding of the passage, we
can say it is directed towards the audience of
astronauts.
Ask doubt with Question Id: 2382
59)c; From the first paragraph, we can say, author has
used familiar objects like an ill-spun top and reference
to geometrical forms like cone. Hence, option-c is
correct.
Ask doubt with Question Id: 2383

Logon to www.CampusRecruitment.co.in and ask doubt with Question Id. 3.76


CLOSET - FILL IN THE BLANKS
CONCEPTS 9. If you dont spot any signal words or keywords and
A cloze test, consisting of a portion of text with certain if you dont know the meaning of the option words,
words removed, where the student is asked to replace read once again and choose the one that sounds the
the missing words from among the given choices. best, eliminating one or two choices.
The sentence completion section tests your vocabulary
skills as well as your reading ability. The question A Solved example is given below
contains a paragraph or a sentence expressing a (A) ____ of his reputation as a comedian, the director of
complete idea that can be understood without any the film ____ Mr.Bean from acting a very serious role of
additional information. Each blank need to be filled up a priest in his new movie.
appropriately retaining the meaning of the sentence Options:
and the syntax. 1. a) Since b) Due to
Strategies to solve questions on Sentence Completion. c) Because d) As
First, slowly read all the text without filling any of the 2. a) Encouraged b) Discouraged
gaps. Read it until you got a clear understanding of c) Supported d) Boosted
what the text is about. Explanation: Among the given choices, 'because' is the
1) First, complete the gaps you are absolutely sure of. only word that can go with 'of'. Similarly, we can
2) Next try and find out what the missing words in the understand that it is difficult for a comedian to act as a
remaining gaps are. See which part of speech may fit in priest. So it is natural that the director of the movie
each gap (article?, pronoun?, noun?, adverb?, 'discouraged' him. Therefore, 'discouraged' is the most
adjective?, preposition?, conjunction?, verb?) and pay suitable word.
special attention to the grammar around the words in EXERCISE-I
each gap. (A) A blog is a web page made up of brief, frequently
3. Read the sentence once again after choosing the updated entries that are arranged (1) like a journal.
words to fill up the blanks. The purpose of blogs (2) greatly from links to news,
4. To solve the sentence completion section, you must photos, even fiction. Blog posts are (3) to instant
have a through understanding of the sentence given. messages to the web. Many blogs are (4) "what's on
This understanding is possible only if you develop my mind" type musings others are collaborative efforts
your knowledge of the root words, synonyms, based on a (5) topic or area of mutual interest.
antonyms, analogies, idioms, phrases etc.
5. In many cases, several options may fit in but you 1. a) symmetrically b) chronologically
must select the one that gives the meaning of the c) interestingly d) passionately
sentence most precisely. 2. a) depend b) shift
6. Understand the message of the sentence by c) vary d) change
analyzing the principal clause(s) and the sub-ordinate 3. a) familiar b) similar
clause(s). Briefly speaking, analyze whether the c) unique d) superior
sentence is a simple sentence or compound sentence or 4. a) personal b) ephemeral
complex sentence. c) temporal d) local
7. Observe the subject of the sentence, the keywords or 5. a) vague b) specific
the signal words. For example the key words like, c) controversial d) contemporary
therefore, 'so', 'because', 'although', 'in addition to', 'further
more' etc can help you to make the right option from (B) My final year at MIT was a year of (6) . A new
the options given. wave of thought (7) through the country in those
8. Understand the authors tone. It means whether the years. The popular view in those days was that a belief
author is positive or negative in his/her approach to the in scientific methods was the only (8) approach to
idea he has presented. Look for negative words like knowledge. If so, I wondered, what about spiritual
'no' or 'not'. Negative words can change the direction (9) ? I had been taught from my early childhood that
of the sentence. knowledge could be (10) only through the inner
experience.

Logon to www.CampusRecruitment.co.in and ask doubt with Question Id. 3.77


Find the details below to purchase the book.

How to order
Click Here

Click Here

Click Here

To purchase at us Click Here

www.CampusRecruitment.co.in
For Details, contact: sales@campusrecruitment.co.in
Or call us at: 07032206275
16)b; Here if we choose the option-d then 'Antarctica 25)d; Strange behavior is the correct usage. All other
adventure' is somehow meaningful but 'serious nearing' options are irrelevant in the present context.
is meaningless. Option-a and c both does not fit with Ask doubt with Question Id: 2056
its preceding proper noun and adjective respectively. 26)a; The sentence has negativeness therefore we have
Expedition means a journey undertaken by a group of to pick up the words with negative sense. Only one
people for the purpose of research or exploring option-a has negative sense. Hence is the correct
something. Voyage also means a journey involving choice.
travel by sea. But the purpose of voyage may not be a Ask doubt with Question Id: 2057
research or an exploration. 27)d; Deprive means lacking a specified benefit.
Ask doubt with Question Id: 2046 Performance of people is effected by the lack of sleep.
17)c; As the given sentence is a disciplinary Ask doubt with Question Id: 2058
exaggeration regarding 'science'. Therefore you should 28)d; Depicts means to describe. And predicament means
have an idea before selecting the option that the author unpleasant. These two words are the appropriate
must adopt figurative words to express his choices. Delineates also means to describe. But the word
exaggeration. So the related meaning sticks only in the changing is not the best choice in the context.
option-c. Ask doubt with Question Id: 2061
Ask doubt with Question Id: 2047 29)d; Distance education is the correct usage than
18)b; In the second blank the words '____ and skilled' is individualized education or remote education.
used. Intricate which means very complicated, is the Ask doubt with Question Id: 2062
correct choice. No other word is relevant for this blank 30)c; The first blank should take V3 form because the
from among the given options. Incessant means never sentence is in passive voice. Only option-c and d have
ending. Exorbitant means excessive. V3 form. The word 'succumbed' diverts the theme of the
Ask doubt with Question Id: 2048 sentence. So, only option-c is correct.
19)a; Proposal means presentation for an action. So, it is Ask doubt with Question Id: 2064
the appropriate choice for the first blank. Blueprint
means a plan or model or design. It is the correct choice
for the second blank.
Ask doubt with Question Id: 2049
20)b; Driving force means motivation. All other options
are not the best choices.
Ask doubt with Question Id: 2050
21)d; Blend means combination of; Synthesis also means
combining. But the word arena does not fit for the
second blank. Expectations is the correct choice for
second blank.
Ask doubt with Question Id: 2051
22)c; The word accessible only makes sense in the first
blank. Immerse means involve oneself deeply in a
particular activity.
Ask doubt with Question Id: 2052
23)b; The sentence speaks about the free press. Hence,
democratic is the correct choice for second blank. The
word democratic gives a sense of free or freedom.
Ask doubt with Question Id: 2054
24)c; You may confuse between option c and d.
Intrigued means attracted. But as the sentence is
speaking about the president, humility is the most
appropriate word than simplicity. Humility means
humbleness, not proud or arrogant.
Ask doubt with Question Id: 2055
Logon to www.CampusRecruitment.co.in and ask doubt with Question Id. 3.83
SENTENCE REARRANGEMENT
CONCEPTS all, to begin with, consequently, considering, I mean, in
Sentence Rearrangement as the name itself advocates, other words, as a result, last of all, in summary, in
consist of sentences not arranged in a logical sequence. conclusion.
A choice of arrangement of the sentences is given from Abbreviations or Acronyms: If full form and its
which the candidate has to choose the most logical abbreviation or acronym are present in two different
sequence which would be the most appropriate for sentences, then the sentence containing the full form
conveying the message of the passage. will come before the sentence containing abbreviation
Type of Questions: or acronym.
Four Sentences: In this type, paragraph consists of four Personal and demonstrative Pronouns: If a sentence
sentences which are jumbled and the student is contains a personal or demonstrative pronouns (i.e.
supposed to choose the correct sequence. you, your, he, she, it, they, this, that, these, those etc)
Five Sentences: This type is quite similar to the "four without mentioning the person, place or object it is
sentence" except instead of four sentences student has referring to, the person, place or object must have come
to rearrange five sentences. in the previous sentence.
Six Sentences: This is a typical one of this chapter Checking vocabulary inventiveness
where six sentences are given in which first and sixth For this process a candidate has to look into the
sentences are fixed. The student has to rearrange the starting and the concluding words of the sentences that
four sentences in between the first and sixth. may have an apparent link. In addition to this students
Tips to solve: have to concentrate in some particular conjunction
Read as they are: It is a kind of reading which gives words that may appear either ending or starting of the
an overall concept to the reader. In this step student has sentences, namely, so, therefore, moreover, meanwhile,
to tick or write the crucial words to make his nevertheless, notwithstanding etc.
remembrance more effective. He has to have an idea of CONCEPTUAL EXAMPLES
passage by this squashed reading technique. Rearrange the following sentences (A), (B), (C), (D) in
Finding either starting or concluding sentences the proper sequence to form a meaningful paragraph,
Finding either starting or concluding sentences is also then answer the questions given below them.
necessary to get the answer properly. In this finding 1) A. In formal speech, syllables are likely to be more
procedure students have to keep an eye in the options deliberately sounded than in informal speech
too. Therefore finding either starting or concluding B. Yet dictionary editors have no choice but to deal
sentences can be derived as technical method of logical with each word as an individual entity.
answer. C. The pronunciation of words is influenced by the
Linking sentences: This is a technique of mastering situation.
in jumbled paragraph. In order to get proper linking D. Further, the pronunciation of a word is affected by
sentences students have to identify the main or its position in the sentence and by the meaning it
supplementary ideas which constitute the message carries.
being conveyed by the paragraph. If a clear picture of a) ACBD b) ACDB c) ABCD d) CADB
the main paragraph is found, then the link of the Explanation: First of all, try to identify the starting and
sentences will be come out automatically. Once the link ending/ concluding sentences. Sentences B, D can not
is found, then obviously the given options will direct be the first sentence as they have the linking words like
the correct answer. 'yet', 'another' which generally indicates continuation
Here is the list of words which are used as linking to its previous sentences. Sentence-A is speaking about
devices: two kinds of pronunciation. But sentence-C has an
also, again, as well as, as a rule, besides, furthermore, introduction about pronunciation. Hence, it forms the
generally, in addition, likewise, moreover, consequently, first sentence. Sentence-A, D are continuation for C as
similarly, to sum up, hence, otherwise, subsequently, later, they explain about how the pronunciation is affected.
therefore, thus, for example, for instance, to illustrate, much Ask doubt with Question Id: 1947
like, such as, above all, besides, even though, although,
despite, probably, due to, unless, whether, until, yet, first of

Logon to www.CampusRecruitment.co.in and ask doubt with Question Id. 3.84


Find the details below to purchase the book.

How to order
Click Here

Click Here

Click Here

To purchase at us Click Here

www.CampusRecruitment.co.in
For Details, contact: sales@campusrecruitment.co.in
Or call us at: 07032206275
11) Out of all three options BCD no one has linking Similarly C is supporting D with an explanation of the
quality with 1. A is rightly fit after 1 because the phrase same process. B keeps the topic one step ahead with a
'after the partition of the Indian subcontinent' shows subordinate clause 'In its original version'.
the link with 1 about the regional integration in South Ask doubt with Question Id: 2275
Asia and the liquidation of the British Empire. B 17) Starting sentence 1 explains about 'visual
explains the extension of the meaning about India, recognition' and the circumstances of involvement and
Pakistan. D holds the content of the total talk hence it has a direct link with C, because C points out the
comes after B. controversy surrounding the question of 'visual
Ask doubt with Question Id: 2270 recognition' the prime matter of the sentence 1. Then A
12) Here the first link is between 'the demand for is supporting C because it is explaining about the step
petroleum product' of D with 'commercial energy of the Psychologists regarding the said matter.
consumption' of 1. Similarly A holds the continuity 'the Moreover 6 encompasses D to its circle because of its
demand, for petroleum' in same manner. C is a direct linking meaning 'internal recognition'. Therefore we
explanation of D and A. Even in the given answers have the sequence CABD.
there is only one option that starts with this sequence Ask doubt with Question Id: 2276
DAC. 18) B explains about 1 because 1 is about the history of
Ask doubt with Question Id: 2271 mammals and B explains it's evolutional changes. D
13) The whole matter of talk persuades the meaning of spells out the sequential explanation after B about
'the success of any unit in a competitive environment' 'average size', 'larger mental capacity'. C excels the
and there is a co-relative sentence that signifies 'cause topic into a progressive way So C comes after D.
for poor generation of surpluses' it's C and its added Similarly A follows C.
meaning covers at B. Again D excels 'the lack of a Ask doubt with Question Id: 2277
mechanism in Indian tax Laws' that directly relates to
B.
Ask doubt with Question Id: 2272
14) The present sentence 1 is based on 'Count Rumford'
depicting link at A as 'He' initiates subjective pronoun.
However second initiation of the topic starts at C
expressing another personality and his work
'Lavoisier'. So C comes after A. Again an apparent
explanation of the starting sentence about 'Rumford' is
clearly visible at D. Similarly B maintains the
remaining sequence.
Ask doubt with Question Id: 2273
15) A has a direct link with 1 because 1 expresses that
the death of cinema and A states that never happened.
Similarly B comes after A because B accentuates a
meaning regarding the killing of cinema by the
television. Then D supports B because D tells about
home computer as its projecting nemesis.
Ask doubt with Question Id: 2274
16) In sentence rearrangement we need to arrange the
sentences in a meaningful and a sequential order, only
then the standard of language will be focused. Here if
we take D first in our sequential sentence order then it
can create an appreciable order of meaning
undoubtedly. Sentence 1 is telling about 'the idea of a
sea-floor spreading' and 'the theory of plate tectonics'
that is directly relating with D, where D is telling the
formulation of' the sea-floor spreading hypothesis'.
Logon to www.CampusRecruitment.co.in and ask doubt with Question Id. 3.99
Find the details below to purchase the book.

How to order
Click Here

Click Here

Click Here

To purchase at us Click Here

www.CampusRecruitment.co.in
For Details, contact: sales@campusrecruitment.co.in
Or call us at: 07032206275
PLACEMENT PAPERS
PLACEMENT PAPER1
QUANTITATIVE APTITUDE 11) At what time between 2 O'clock and 3 O'clock the
1) A train running at a speed of 90 km/hr crosses a two hands coincide?
platform double of its length in 36 sec. What is the 10
a) 10 minutes past 2 O'clock
length of platform in meters? 11
a) 450 b) 200 c) 300 10
b) 11 minutes past 2 O'clock
d) Can't be determined e) None of these 11
2) 'A' can finish a work in 32 days and 'B' can do the 10
same work in half the time taken by 'A'. Then working c) 12 minutes past 2 O'clock
11
together what part of the same work they can finish in 10
a day? d) 13 minutes past 2 O'clock
11
3 1 5 16 1 e) None of these
a) b) c) d) e)
32 8 32 32 32 12) Find the mean proportion of 45 and 405.
3) The owner of an electronic shop charges his a) 9 b) 15 c) 90 d) 135 e) None
customer 24% more than the cost price. If a customer 13) Find the cost of running a fence round a square
paid Rs 10080 for a TV set, then what was the cost price field 49284 m2 in the area of Rs.3 per meter.
of the TV set? a) 2614 b) 2714 c) 2914 d) 2664 e) None
a) Rs 8119 b) Rs 8129 c) Rs 8250 Directions (14 to 17): Following table gives the
d) Rs 8139 e) None of these production of computers of six companies A, B, C, D,
4) What would be the simple interest obtained on an E and F over the years.
amount of Rs 12690 at the rate of 6% per annum for 3 Production of Computers (in 000's)
years? Years 2005 2006 2007 2008 2009 2010
a) Rs 2423.40 b) Rs 2233.40 c) Rs 2284.20 Company
d) Rs 2525 e) None of these A 125 114 85 95 138 146
5) Find the compound interest on Rs 8000 at 5% per B 102 90 72 88 115 145
annum for 3 years compounded annually? C 80 116 110 80 84 75
a) Rs 1261 b) Rs 6261 c) Rs 9261 D 68 156 142 60 95 84
d) Rs 8261 e) None of these E 100 120 126 115 95 80
6) 6 same type of machines can produce a total of 270 F 140 110 126 124 90 88
bottles per minute. How many bottles are produced by 14) What is the ratio of the total production of
the 10 machines in four minutes of time? companies B, C and E together in 2006 to the total
a) 648 b) 1800 c) 2700 d) 10800 e) None production of companies A, C and D together in 2008?
7) Find the least perfect square number which is a) 10 : 11 b) 11 : 10 c) 27 : 31
divisible by 8, 15 and 24. d) 55 : 56 e) None (Asked in Infosys)
a) 3600 b) 360 c) 6400 d) 64 e) None 15) What is the approximate percentage decrease in
8) A person had a rectangular shaped garden with production of computers of company D from 2006 to
sides of 16 feet and 9 feet. The garden was changed into 2009?
a square with the same area as the original a) 10% b) 90% c) 40% d) 45% e) 30%
rectangular shaped garden. How many feet in length 16) What is the average production of computers of
is each of the sides of the new square shaped garden? company E over the year?
a) 7 b) 9 c) 12 d) 55 e) 16 a) 106 b) 10000 c) 132 d) 106000 e) None of these
9) The average of five consecutive numbers A, B, C, D 17) The production of F in 2010 is the same as the
and E is 48. What is the product of A and E? production of B in the year,
a) 2162 b) 2208 c) 2024 d) 2300 e) None a) 2009 b) 2006 c) 2008 d) 2005 e) None of these
10) A, B and C enter into a partnership. A contributes 13
Rs. 2400 for 6 months. 'B' contributes Rs.7200 for 2 18) The sum of a number and its reciprocal is .
6
months and 'C' contributes Rs.3500 for 7 months. If the Find the numbers?
total profit be Rs.1250, what is A's share in the profit? 3 2 1 13 1 13
a) 121 b) 112 c) 111 d) 122 e) None a) , b) , 3 c) 6, d) , e) None
2 3 3 6 13 6
Logon to www.CampusRecruitment.co.in and ask doubt with Question Id. 4.1
Find the details below to purchase the book.

How to order
Click Here

Click Here

Click Here

To purchase at us Click Here

www.CampusRecruitment.co.in
For Details, contact: sales@campusrecruitment.co.in
Or call us at: 07032206275
PLACEMENT PAPER5
QUANTITATIVE APTITUDE 8) Which of the statements is true for 3111 & 1714 ?
11 14
Directions(14): Study the following graph carefully a) 3111 is greater b) 3111 is lesser c) 31 17
11 14
to answer the questions that follow d) 31 17 e) None
Monthly income (Rupees in thousands) 9) The average of five numbers is 281. The average of
40
the first two numbers is 280 and the average of last two
35 numbers is 178.5. What is the third number?
30 a) 488 b) 336 c) 228 d) 464 e) None
25 10) At 30 minutes past 4 O'clock, what is the angle
20
between the two hands?
a) 450 b) 480 c) 500 d) 400 e) 550
15
1 1
10 11) Two equal glasses are respectively and
3 4
5 full of milk. They are then filled up with water and the
0 contents are mixed in a tumbler. Find the ratio of milk
2005 2006 2007 2008 2009 2010
and water in the tumbler. (Asked in Syntel)
Arjun Suman Jyothi a) 1 : 5 b) 7 : 17 c) 3 : 9 d) 2 : 5 e) 4 : 7
Monthly income (Rupees in thousands) of three 12) A, B & C invested Rs.500, Rs.630 & Rs.700. If A gets
different persons in six different years. Rs. 75 as profit then how much C will get?
1) What was the difference between the total monthly 3
a) 57 % b) Rs. 105 c) Rs. 90 d) Rs.126 e) Rs.117
salary of Arun in all the years together and Suman's 4
monthly income in the year 2007 ? 13) How long will a man take to go, walking at 4 km
a) Rs. 1.24 lakh b) Rs. 1.14 lakh c) Rs. 11.4 lakh per hour, twice round a circular garden of 70 m radius?
d) Rs. 12.4 lakh e) None a) 13 m 2 sec b) 12 m 13 sec c) 13 m 11 sec
2) What is the ratio of Arun's monthly income in the d) 12 m 11 sec e) 11 m 13 sec
year 2006, Suman's monthly income in the year 2007 Directions(14 to 15): Read the following question and
and Jyothi's monthly income in the year 2005? the conclusions that follow and answer as
a) 6 : 3 : 5 b) 6 : 4 : 5 c) 5 : 6 : 4 d) 5 : 4 : 7 e) None a: If statement1 alone is sufficient, but statement2 alone is
3) In which year was the difference between Jyothi's not sufficient to answer the question.
and Arun's monthly income the second highest? b: If statement2 alone is sufficient, but statement1 alone
a) 2005 b) 2006 c) 2007 d) 2009 e) 2010 is not sufficient to answer the question.
4) The monthly income of Suman in the year 2009 was c: Both statement1 and 2 together are sufficient to answer
approximately what percentage of the monthly income the question, but neither of the statement alone is sufficient
of Jyothi in the year 2010? to answer the question.
a) 72 b) 89 c) 83 d) 67 e) 95 d: If both statements alone are sufficient to answer the
5) How many ways are there to lay four balls, colored question.
red, black, blue and green in a row? e: If statement1 and 2 together are not sufficient to answer
4! the question asked, and additional data specifics to the
a) 4 b) 4 ! c) 44 d) 4 e) 4(4 !)
4 problem are needed.
6) There are 20 states in a certain country and every 14) Is y is a positive number?
pair of them is connected by a road way. How many Statement1: 2x+y > 27; Statement2: x3y < 24
road ways are there? (Asked in ABB) 15) If x is a positive integer less than 30, is 'x' an odd
20.19 number?
a) 20! b) 19! c) 18! d) e) 20
C2
2 1: When x is divided by 3, the remainder is 2.
7) Two cards are selected at random from 10 cards 2: When x is divided by 5, the remainder is 2.
numbered 1 to 10. Find the probability that the sum is 16) A train of length 330 meters crosses a platform of
odd if two cards are drawn together. length 550 meters in 44 seconds. What is the speed of the
4 5 5 5! 2 train?
a) b) c) d) e)
9 9 10 10 ! 10 a) 5 m/s b) 7.5 m/s c) 12.5 m/s d) 20 m/s e) None
Logon to www.CampusRecruitment.co.in and ask doubt with Question Id. 4.43
Find the details below to purchase the book.

How to order
Click Here

Click Here

Click Here

To purchase at us Click Here

www.CampusRecruitment.co.in
For Details, contact: sales@campusrecruitment.co.in
Or call us at: 07032206275
68)d; Nowhere in the passage it is mentioned that they 78)c; Paucity means scarcity, rarity, shortage and
are of lackadaisical (means arrogant, apathetic, dearth. Abundance, meaning wealth, large quantity is
careless etc.,) or selfish; nor they have any authority, the opposite word of paucity.
whatsoever. If they really have bureaucratic authority, Ask doubt using Question Id: 6314
they would not be middle class people. 79)b; Conclusive means convincing, irrefutable,
Ask doubt using Question Id: 6304 indisputable and unquestioning.
69)a; To hail means to praise and acclaim and this is Ask doubt using Question Id: 6315
contrary to what has been proposed in the passage. 80)a; When something is manifested, it is very clear,
Ask doubt using Question Id: 6305 visible, evident and noticeable. The opposite word
70)a; The very opening sentence of the passage speaks would be conceal.
about the semantic conspiracy which means linguistic Ask doubt using Question Id: 6316
intrigue; a trickery usage of language. 81)c; Impudent, disrespectable, brazen, insolent and
Ask doubt using Question Id: 6306 rude are a few synonyms of the underlined word
71)a; The paragraph can start only with the sentence E. impertinent.
The opening words of the sentence of B these two Ask doubt using Question Id: 6317
parties is a continuation of the last words of the 82)c;Deprecating somebody is denouncing,
sentence E two parties. There are only two options condemning, denigrating and protesting.
with EB; in the option 4, D follows B; but whom the Ask doubt using Question Id: 6318
opening pronoun he of D does refer to? F is the 83)c;Commodious means spacious, large, voluminous
further explanation of B. Hence EBFCAD. and extensive.
Ask doubt using Question Id: 6307 Ask doubt using Question Id: 6319
72)a; Excepting B, the rest of the sentences can be the 84)d; if someone is confounded means he is puzzled,
beginners of a paragraph. Excepting in the first answer confused, bewildered stunned and baffled.
choicea, sentence B breaks the sequence of the rest of Ask doubt using Question Id: 6320
the choices. 85)a; To be undaunted is to be impervious, undisturbed
Ask doubt using Question Id: 6308 and fearless.
73)b; It is only in the second option we find the Ask doubt using Question Id: 6321
sentence A in continuation with the opening sentence
D. Sentence A alone explains, why the fear has such
enormous effect on the human beings.
Ask doubt using Question Id: 6309
74)c; In the first, second and fourth answer choices,
sentence D breaks the sequence as there is no reference
to the opening words of the sentence D this image in
the preceding sentence; only in choicec, D is preceded
by B.
Ask doubt using Question Id: 6310
75)b; Sentence B can only be a concluding sentence; we
find in the answer choiceb, a sequence of the
sentences logically leading to the concluding sentence
B.
Ask doubt using Question Id: 6311
76)b; To yield is to surrender, submit, and obey
someone unconditionally. 'Resist which means defy,
oppose and refusing to accept is the opposite word.
Ask doubt using Question Id: 6312
77)c; Excepting seldom which means rarely,
infrequently and not often, is the antonym.
Ask doubt using Question Id: 6313

Logon to www.CampusRecruitment.co.in and ask doubt using provided Question Id. 4.54
TECHNICAL INTERVIEW
TECHNICAL INTERVIEW
In the present days, most of the companies are conducting the Technical Interview Round in their recruitment
process. It is one of the efficient way of filtering the suitable person for the industry. Many students have a wrong
notion that it is very hard to clear this round. But it is not true. This round will mainly focus on the basics or
fundamentals of the stream. Students need to be thorough and confident in the fundamentals of the subject.
Recruiting team will not expect that the student should answer all questions perfectly, but he/she must be
reasonably good and confident about the subject. The most important questions for technical interview are given
below. These questions will give you the idea about what type and toughness generally the companies are
asking.
CIVIL ENGINEERING
Building Materials and Construction 19) Up to what spans the brick lintels are used?
1) What is the term used to call the vertical member in 20) Can you define Wainscot which is used in paneling
the middle of the door/window frame? of wood masonry wall?
2) What is the standard or commonly recommended 21) Can you draw a sketch showing soffit, tread and
depth between finished level of ground and the general rise of a typical stair case?
ground level around the building? 22) What is the minimum percentage of the window
3) Do you know the number of BIS standard used for area is provided with reference to the total inside area
building drawing purpose? of the room?
4) Can you draw a typical sign indicating brick? 23) What is the commonly used thickness of the
5) State a few conditions at where eccentricity of plywood facing on flush door?
building occurs? 24) What is the term used to indicate the angle formed
6) What are the requirements of a material used for at the intersection of the two roof slopes?
damp proofing in building construction? 25) Steel trusses are generally adopted because ............
7) At what level damp proofing course on the internal state the reasons?
wall is provided if two ground floors at different levels 26) What are the factors that influence quality of
are connected by an internal wall? mosaic tiles/flooring?
8) What is the term used to indicate the sides of the 27) Where the external metal staircase is generally
openings such as doors or windows? used?
9) What is the maximum thickness of mortar joint 28) What is the most commonly adopted values of
width provided in Ashlar fine masonry? tread and rise, for Indian conditions?
10) What is the term used to indicate the rod which is 29) What type of lines a combined line is represented
used to dress roughly the hard stone? by?
11) What is the bond that is provided to strengthen the 30) State the significance of GTS bench mark?
corner of a wall where a modified form of English bond Transportation Engineering
is used? 31) What is the camber provided in case of WBM
12) What is the duration of immersing brick which are roads?
used before they actually placed in position, during 32) What is the standard interval of providing
construction? expansion joints in a CC pavement slab?
13) Does an expansion joint in brick wall is necessary? 33) What is the minim grade of concrete recommended
State its interval along its length? in case of truck serving CC pavements?
14) What is the minimum depth of concrete at the 34) Do you know anything about Superpave
crown of a jack arch roof? technology?
15) What material is used to obtain noiseless flooring? 35) What do you know about CRF: Central Road Fund?
16) What is the approximate thickness of brick course 36) What is the finding scenario of NHDP?
used in Madras Terrace Roof? 37) What is the funding difference between PMGSY an
17) What is the term used to indicate inner surface of d Bharath Nirman Projects?
an arch? 38)What are the instruments used during
18) What is the commonly adopted bearing length used reconnaissance survey?
for wooded lintels?
www.CampusRecruitment.co.in 5.1
Find the details below to purchase the book.

How to order
Click Here

Click Here

Click Here

To purchase at us Click Here

www.CampusRecruitment.co.in
For Details, contact: sales@campusrecruitment.co.in
Or call us at: 07032206275
228) Draw zero sequence networks of loads 247) Draw the phasor diagrams of synchronous
connections for star ungrounded neutral, star generator feeding constant active power into infinite
grounded neutral, star grounded neutral with an bus bars as excitation is varied.
impedance Zn and delta connected loads. 248) What do you understand about insulation
229) Explain formulation of Ybus using nodal method coordination, protective margins and protective ratio?
249) Explain the N-R method of load flow studies.
and singular transformation method.
Explain the reasons for the order of Jacobian in N-R
230) Define sparsity. What is the value of sparsity of
method using polar coordinates is smaller than in N-R
Ybus for a practical power system?
method using rectangular coordinates.
231) What are the assumptions made to develop
250) What is traveling wave? Explain the concept of
relation between bus voltages and bus currents in load
standing wave and voltage standing wave ratio.
flow studies?
232) Define voltage regulation and efficiency of a
transmission line. Explain the effect of load power
factor on regulation and efficiency.
233) Draw phasor diagrams and write A, B, C and D
parameters for nominal T and circuit of transmission
line.
234) Derive A, B, C and D parameters for nominal T
and methods.
235) Explain how to determine A, B, C and D
parameters of a transmission lines practically.
236) Explain how to determine +ve, -ve and zero
sequence impedances of transformers practically.
237) Explain how to determine +ve, -ve and zero
sequence impedances of alternators practically.
238) Explain phenomenon of current chopping in
vacuum circuit breakers. State the properties of contact
materials used for vacuum interrupters.
239) Define switchgear and differentiate between low
voltage switchgear and high voltage switchgear.
240) Define coefficient of earthing and state the
significance of it in selection of voltage rating of surge
arrester.
241) Explain about single frequency transient and
double frequency transients.
242) Explain the working principle of current limiting
reactors and explain why we need these reactors in
power systems.
243) Explain the principle of directional element. Draw
and explain impedance relay characteristics with
directional element.
244) What do you understand about power swings and
explain its affects on distance relays.
245) Define reach and under-reach of distance relay.
Explain the effect of arc resistance in distance relays.
246) Draw voltage characteristics of surge arrester and
define residual voltage, impulse spark over voltage and
power frequency voltage of a surge arrester.

www.CampusRecruitment.co.in 5.17
COMPUTER SCIENCE AND ENGINEERING
C Programming }
1) Write a program to print? }
***** 9) What is the difference between
**** a. parameter passed by reference
*** b. parameter passed by value
** 10) In the code below, which variable has the largest
* scope?
2) When should a type cast be used? include <stdio.h>
3) What are the different data types? int b;
4) What is the output of the below program? int main()
#include<stdio.h> {
int main(){ int c;
char a='A'; return 0;
printf("size of variable a is %d \n",sizeof(c)); }
printf("size of A is %d ",sizeof('A')); int a;
return 0; 11) Can we convert int to double? If yes, give an
} example expression?
5) Does the below program runs successfully? 12) What is the difference between string and character
#include<stdio.h> arrays?
int main(){ 13) Define structure and give an example?
int const a = 1; 14) What does static variable mean?
a = 2; 15) What is the benefit of using 'const'?
} 16) What is recursive function? Write a program of
6) What is difference between i++ and ++i? Fibonacci series using recursive function?
7) What is the output of the below program? 17) Give a syntax of declaring array?
include <stdio.h> 18) Write a program to print every element of 2 x 5 x 3
int main() dimensional array?
{ 19) What is a pointer?
int m=40,n=20,o=20,p=30; 20) How do you use a pointer to a function?
if (m>n && m !=0) 21) What do you mean by inline function?
printf("&& Operator:Both conditions are true\n"); 22) What are the different storage classes in C?
if (o>p || p!=20) 23) What is the difference between printf ( ) and sprintf
printf("|| Operator : Only one condition is true\n"); ( )?
if (!(m>n && m !=0)) 24) Does there exist any other function which can be
printf("! Operator : Both conditions are true\n"); used to convert an integer/float to a string?
else 25) Can a variable be both constant and volatile?
printf("! Operator : Both conditions are true. But, 26) Advantages of a macro over a function?
status is inverted as false\n"); 27) Which bit wise operator is suitable for checking
} whether a particular bit is on or off?
8) What is the output of the following code? 28) When should the volatile modifier be used?
#include<stdio.h> 29) const char *p , char const *p
void main(){ What is the difference between the above two?
int check=2; 30) What is output of below program?
switch(check){ #include <stdio.h>
case 1: printf("Infosys"); int main(){
case 2: printf("Gail"); int a=500;
case 3: printf("L&T"); char *ptr=(char *)&a;
default: printf("AirIndia"); printf("%d\n",*ptr);

www.CampusRecruitment.co.in 5.22
Find the details below to purchase the book.

How to order
Click Here

Click Here

Click Here

To purchase at us Click Here

www.CampusRecruitment.co.in
For Details, contact: sales@campusrecruitment.co.in
Or call us at: 07032206275
217) Give me some scenarios where document 245) Assume you are developing a web application for
object being used? travel portal, what are the requirements you want to
218) What is difference between Prompt and Alert gather?
methods in JavaScript? 246) What is difference between white-box testing and
219) What is XML and give the example of XML? black-box testing?
220) Why XML is used in web technologies? 247) What is system testing, When does system testing
221) Design XML for the below given scenario can be conducted?
A shopping floor have multiple racks, each rack has 248) List some golden rules for designing UI?
multiple shelfs, each shelf has multiple products, each 249) What is UML? Explain some symbols which are
product has price and manufacturing date. used in UML? List some tools for drawing UML
222) What is the purpose of DTD and where it is diagrams?
mentioned in the XML document? 250) What is Use case and What is Test Case? What is
223) What DOM stands for? And what is relation difference between both?
between DOM and XML? 251) List some of the risks that software systems may
224) What are two parsing mechanisms of XML? encounter?
225) How do we set and get the properties to Java bean Project Interview Questions
class? Give the example java bean class? The following are some of the commonly asked
226) What is default port of tomcat web server? interview questions on your project work.
227) In which folder of tomcat do we deploy web 1) Tell me what you learned from your project?
applications? 2) Why did you choose this project?
228) Explain the life cycle of the servlet? 3) What were the objectives of the project?
229) In how many ways can we track the session? 4) What would happen if ____?
230) Write a servlet, which displays all the form 5) What are the limitations of your project?
attributes? 6) If you were to start your project again, is there
231) What is difference between Session Variable and anything you would like to change?
Application Variable? 7) What were the best features of your project?
232) What is filters and when does filter gets executed? 8) Is there another possible explanation for your
233) What is deployment descriptor, What is the name results?
given to deployment descriptor file? 9) What further research would you liked to have
234) Name the implicit Object provided by JSP? conducted, and why?
235) What is difference between forward and redirect 10) How could it help people?
methods? 11) Can it be implemented in real time?
236) Give the syntax for JSP declaration and JSP 12) Working of all individual components.
expression? 13) What is a project?
237) How many types of statements are there in JDBC. 14) Name five signs that indicate your project may fail.
What are they? 15) Tell us about a project in which you participated
238) What is meant by JDBC driver and why it is used and your role in that project.
for? 16) Tell us about any creative idea or innovative
239) Give the code snippet to open connection object solution you brought to a project.
using the JDBC? 17) What types of tools are involved in your project?
240) Give the syntax which iterating the Result Set 18) What is the team size of your project?
Object?
Software Engineering
241) What CMMI stands for?
242) Mention the different stages in SDLC?
243) Explain how water-fall model works for software
development?
244) What does software requirement document
consists?

www.CampusRecruitment.co.in 5.29
SOFTSKILLS
and
JOB SEARCH/APPLYING TIPS
HR INTERVIEW
The interview is the last step for hiring and probably Learnability is another sign of a good leader. Letting
the most important. Interviews are conducted to assess the interviewers know your eagerness to learn new
a candidates suitability for an organization and the things will prove to be a bonus for you.
hiring role. f) Emotional Maturity: A candidate must be calm and
Why an interview is conducted: composed during an interview. Sometimes, the
The purpose of an interview is to ascertain what a interviewer, on purpose may provoke you by asking
candidate has mentioned in his resume. It is a brief some unexpected questions. Do not get provoked or
meeting where your technical know-how will be angry. Instead, show your maturity by answering the
assessed in addition to your communication skills, question in a manner that does not offend the
motivational factors, your attitude, your goals and interviewer. It is better to be honest and give direct
objectives. An interview is a two way process where an answers instead of speaking as if you agree with all the
employer gets to know the prospective( to be hired) points that the interviewer says. The given situation
employee and an employee gains insight of the should be handled with ease, maturity and composure.
organizational and the role being offered to him. g) Dressing Sense: It is important to dress right for an
What qualities does an interviewer observe? interview. Your appearance talks loads about your
a) Attitude: The most important factor that determines personality. Wear neatly ironed formal clothes with a
who gets hired and who doesnt is NOT who is best good pair of shoes. Women should apply minimal
qualified, who has the most experience or skills, or make up and avoid too much of jewelry. Both men and
who has the best resume. It is the attitude! Show women should have well manicured nails and a
positive attitude and your excitement for the job. Do professional hairstyle. Going shabbily dressed for an
not show arrogance even if you are a college topper. interview would cast an impression of your being
b) Communication Skills: Regardless of how much of uninterested towards the meeting.
knowledge you possess, unless you know how to h) Integrity: Every interviewer appreciates honesty as
express yourself clearly, the interview may not turn out that is one of the main traits to look for in a candidate.
to be in your favor. Powerful communication skills Answer the questions honestly. Do not lie about your
have the ability to compensate for short coming in professional achievements. Be humble. Do not throw
other areas. A candidate must speak confidently and an air of arrogance. If you are appreciated for
use clear language with as little jargon as possible. His something, be gracious to say thanks.
conversation with the interviewer must reflect his i) Flexibility: An employer should be convinced about
enthusiasm, awareness and his attitude. your ability to adapt to different situations. Show the
c)Confidence: Approaching an interview with employers that you are flexible and can adjust to
confidence is one of the keys of a winning strategy. different circumstances. If you are asked for example,
Your confidence is a reflection of optimism and speaks your willingness to re-locate far away from your family
a lot about how you would handle a challenging for a project, mention that you would be happy doing
professional situation. Confidence is great; so as your career and organizations interest are
overconfidence is a strict no-no. foremost priorities. How well you adjust to the
d) Body language: Good body language is an indicator situation is a trait employers look for in their
on good non-verbal communication. Walk into the employees.
room with a confident smile and introduce yourself to j) Learnability:
everyone present with a firm handshake. Maintain a Learning is a continuous process. Desire to keep
good posture, do not slouch, maintain eye contact, and learning shows an employer that you are enthusiastic
nod your head when being talked to. Do not yawn, bite about what is being offered to you. Learnability is a
your nails, shake your limbs, look around, look down. key to professional as well as organizational growth.
e) Leadership skills: You may be the best of candidates, but unless you have
A leader always motivates. A leader directs his team a thirst for knowledge, your knowledge repository may
towards success. When you attend an interview, make become saturated.
sure your interviewer knows that you are ready to take
initiatives and help the members of your team.

www.CampusRecruitment.co.in 6.1
Find the details below to purchase the book.

How to order
Click Here

Click Here

Click Here

To purchase at us Click Here

www.CampusRecruitment.co.in
For Details, contact: sales@campusrecruitment.co.in
Or call us at: 07032206275
Mock Interview with Body Language

* When the interviewer offers you a seat at the start of the job interview, sit upright but not too stiffly in your chair. This indicates that you
are comfortable and feeling confident. Hunching down in your chair gives the impression of nervousness and low self-esteem. A sloppy
posture indicates a careless attitude and a lack of energy. Sitting on the edge of your chair can come across as being nervous and tense.
1. Tell us about yourself
Find the details below to purchase the book.

How to order
Click Here

Click Here

Click Here

To purchase at us Click Here

www.CampusRecruitment.co.in
For Details, contact: sales@campusrecruitment.co.in
Or call us at: 07032206275
10. Do you have any questions?

One important key to success is self-confidence. An important key to self-confidence is preparation.


Find the details below to purchase the book.

How to order
Click Here

Click Here

Click Here

To purchase at us Click Here

www.CampusRecruitment.co.in
For Details, contact: sales@campusrecruitment.co.in
Or call us at: 07032206275
Tips to use Naukri.com
Naukri.com is a popular website for searching for jobs. Profile updating in Naukri.com:
This website is now serving as a database to all the A) Profile Summary:
recruiters and HRs who want to recruit candidates. You a. Login to your Naukri.com account. Click on My
can get calls from recruiters if you utilize this site Naukri and then click on View and Update Profile.
effectively. Register yourself with the Naukri.com. b. Then the following screen will be displayed. Now
You need to follow a few tips for updating your profile. Click on Edit at Profile Summary.
c. Profile Summary is a short explanation about your
skills, competencies and your objectives. If you fail to
give an impressive profile summary, the recruiters may
not read your full profile for the required job.

B) Resume Uploading:
a. Click on My Naukri and then click on View and Update Profile.
b. The last tab is Resume as shown in the below screen shot. Click on Edit and upload your latest updated
resume and click on save.

www.CampusRecruitment.co.in 6.26
Find the details below to purchase the book.

How to order
Click Here

Click Here

Click Here

To purchase at us Click Here

www.CampusRecruitment.co.in
For Details, contact: sales@campusrecruitment.co.in
Or call us at: 07032206275
Tips to use Monster.com
Registering in Monster.com as member:
1. Go to www.moster.com, then click on Join us button as shown below,

2. Upon clicking on the Join us button, a screen will and hear applicants, and get a feel for how applicants
be displayed just like as shown in naukri.com web present themselves. It is a resume presented on a video
site. Fill all the details correctly and click on Submit clip of 3 to 5 minutes. This can be said as first part of an
button at the bottom to complete the registration interview which is to introduce oneself. The Video
process. Resume will allow recruiters to do a preliminary
Profile updating in Monster.com screening of the candidates without getting involved in
The profile updating in Monster.com is almost similar the processes of interviews and practically evaluate
to that of Naukri.com. You can follow the same rules how they present their experience and skills. Recruiters
that are explained in Naukri.com to update your will be able search for video resumes in the same way
profile. Monster.com also has a facility to search the as text resumes and also receive the video resumes as
jobs by function, jobs by industry, jobs by location and job applications.
jobs by company. In addition, Monster.com has a new How to submit a Video Resume:
feature of uploading Video Resume. A. Login to your monster.com account. Click on
What is a Video Resume? Submit Resume at the top.
It is the new feature introduced by Monster.com. Job B. Fill all the details correctly. At the end of the page
seekers can now upload their Video Resumes in the you can see the following screen. Here write a good
same way as text resumes. This facility will allow the Resume title as explained in the illustration of Profile
job seeker to personally market oneself by presenting a Updating in Naukri.com. Click on Browse to select
resume and a video that speaks directly to employers. your resume to upload. Then click Submit.
The video resume allows prospective employers to see

3. After clicking on the Submit button, you get a page which asks to enter your Educational Qualification,
Work Experience, Other information and Optional information. Under the Other information heading you
can observe the option to upload Video Resume.

www.CampusRecruitment.co.in 6.31
Find the details below to purchase the book.

How to order
Click Here

Click Here

Click Here

To purchase at us Click Here

www.CampusRecruitment.co.in
For Details, contact: sales@campusrecruitment.co.in
Or call us at: 07032206275
RESUME PREPARATION
Your resume is the most important document that the Update your resume on a periodic basis.
employer would refer to. Hence, it is very important to Mention dates in order.
keep your resume professional and up to date. Include Do not forget the basics, like your name.
your professional details, academic details total work Mention the employers for whom you have worked.
experience, skills, tools and programming languages Mention the companies for whom you have interned
known. Do not lie about your skills and experience. (done your projects).
Resume should be written honestly and carefully. Do Do not use heavy vocabulary or too much of
not prepare a very long resume. A short but to the jargon(technical words).
point resume should be prepared. Do not copy the Get your resume reviewed by a professional.
resume from other sources. Write it on your own Things to avoid when writing your resume
although you may take reference from sample resumes. Lying about past work experience or qualification.
Tips to build an effective resume Committing spelling and grammatical errors.
Choose a good resume format. Making the resume colorful in appearance.
Pick up a font that is easy to read. Fonts such as Including too much of information.
Verdana, Times New Roman and Arial in sizes 10-12 Not using the right keywords.
are conducive for reading. Writing an objective that does not match the job.
Use the right keywords in your resume. Writing old or outdated details.
Use effective titles. Sending a handwritten or poorly photocopies resume
Example: Bad title: Software development. Writing long sentences and paragraphs.
Good title: Developing Java API. Writing about your own goals more than your
Divide your resume into sub sections. abilities to match the job expectation.
Example: One section for details about your work Using I in the sentences.
experience, the second section about your academic Writing information that would make an employer
qualifications, the third about your skill set, the fourth feel that you discriminate (Mentioning your age/
about your personal profile etc. marital status/ number of kids/gender etc).
The most important points should be placed at the Writing about hobbies( Unless your hobby would
beginning of your resume. This should be uniformly contribute to your candidature, do not include it).
followed in the individual sections as well. The most Writing incomplete information.
recent work experience will come first. Writing negative things (Like failure in delivering a
Your contact details including your address project).
telephone number and email ID should be clearly Writing please (Please give me an opportunity to
mentioned in the resume. work for you.)
Use bullet points and short sentences in your Contents of A Good Resume
resume. Do not make the resume boring by giving in a. Personal Details: The most important detail is your
depth explanation or being repetitive. name, mentioned in bold. Your contact information
Avoid writing negative sentences. including email IDs and telephone numbers should be
Write a few sentences about your professional goals. clearly written.
Use your testimonial references for any skill that b. Summary: Writing an objective is a thing of the past.
you would like to highlight. Instead, summarize your experience and skill set in one
Example: winning a technical white paper competition or two effective sentences. This would immediately
at the national level (testimonials included). grab the hiring managers attention. E.g. Five years of
Do not include your age, unless it is specifically asked experience in software quality assurance.
for. c. Past Work Experience: Include the details of your
Do not create a standard resume for all the jobs to be previous employment and briefly mention the
applied to. Restructure your resume depending on the significant projects that you were involved in. Mention
job for which you are applying. the organization name and also the duration of your
Do a spelling and a grammar check before you send work. The order should be starting from recent to past.
your resume.
www.CampusRecruitment.co.in 6.37
Find the details below to purchase the book.

How to order
Click Here

Click Here

Click Here

To purchase at us Click Here

www.CampusRecruitment.co.in
For Details, contact: sales@campusrecruitment.co.in
Or call us at: 07032206275
GROUP DISCUSSION
Group Discussion is a process of selection rather than 3. Factual topics The topics are which may directly or
elimination. Group Discussion refers to the process indirectly affect a person in his day to day life.
where by a topic or a situation is presented to a group Womens reservation bill
of candidates. It is a systematic exchange of ideas and Lokpal bill
information among a group of people. In a Group Status of women in India
Discussion, a topic is given and each member is given Global warming.
about 10-15 minutes to think about the given topic. The 4. Controversial topics Topics in which candidates
members are then allowed to present their views and can take a stance for or against the given topic
opinions to the other candidates in the group. The thereby leading to a bit of disagreement among the
Group Discussion facilitates objective thinking, participating members of the group.
systematic presentation of ideas, interacting abilities, For example,
problem resolution, leadership abilities, ability to take We are becoming too dependent on computers.
initiatives, ability to work within a team, flexibility and Video games contribute to youth violence.
assertiveness skills. Reservation system should be removed.
Why GD is a part of the selection process? 5. Abstract subjects These types of topics generally
The primary reason for conducting the Group include imaginary or hypothetical topics. For example,
Discussion is to evaluate how effectively a candidate What would happen if the Earth would stop
would perform as a member of a team. When more rotating?
than one person is involved in problem solving, ideas What would happen if you were given a chance to
have to be brainstormed, collaborated and a conclusion rule the country?
should be reached such that the organizational goals 6. Case study instead of a topic a case study will be
are met along with creating a win win situation for given in this category. They need to resolve the
every member of the team. Thus apart from situation. The objective the case study is to think about
participation and contribution, team behavior and the situation from different angles. Generally real life
attitude of a person are the traits that a company situations are given in case study. A complex and
would be evaluating in the Group Discussion. problematic situation and information about that
The written test evaluates aptitude in Verbal Ability, situation is given to the group.
Quantitative Ability and Reasoning Ability; These Popular Group Discussion Topics
areas to judge the the analytical, logical, numerical and 1) Are Advertisements Beneficial or misleading?
language abilities but they do not test the personality 2) Balance between Professionalism and Family.
traits or group behavior of the individual. The ability to 3) Can Trade help the poor?
deal with people, individually or in a group, is vital for 4) Demographic Dividend in India.
success in the corporate world. Therefore a group 5) Depreciation of Indian Rupee.
discussion is included in the selection process. A group 6) Is FDI good for India?
discussion allows the selectors to judge the individual's 7) Privatization will lead to Less Corruption.
performance and behavior in a group. Different aspects 8) To survive in the civilized world one needs to be
of group discussion include communication skills- both hypocrite.
verbal and non-verbal, Ability to make decisions and 9) Growth and integrity are poles apart.
co-operate with people. 10)Cricket Has Spoiled Other Streams Of Indian Sports.
Types of Group Discussions 11) We will never be corruption free society.
The classification is based on the type of the topics that 12) Indian Primary Education Is Pathetic.
are chosen for group discussion. The topics could be: 13) India needs more entrepreneurs than managers to
1. Current Affairs-Topics related to current news items face new challenges.
related to business, technology, politics, education and 14) Artificial Intelligence - Will man be ever replaced
social activities etc. by machines?
2. Argumentative Issues- A topic is given to the 15) Role of India in combating terrorism.
candidates that they have to analyze and critically
present their views on the topic.

www.CampusRecruitment.co.in 6.41
Find the details below to purchase the book.

How to order
Click Here

Click Here

Click Here

To purchase at us Click Here

www.CampusRecruitment.co.in
For Details, contact: sales@campusrecruitment.co.in
Or call us at: 07032206275

You might also like